Ari 12 CH 4 Moving Charges and Magnetism

You might also like

Download as pdf or txt
Download as pdf or txt
You are on page 1of 43

- CHAPTER 04

Moving Charges

R
"and Magnetism

SI
Chapter Analysis w.r.t, Lost 3 Yeor's Boord Exoms
The analysis given here gives you an analytical picture of this chapter and will help you to
IT
identify the concepts of the chapter that are to befocussed more from exam point of view.
Number of Questions asked in last 3 years
----------
2015 2016 2017
Delhi I All India Delhi ·All India Delhi All India
H
Very Short Answer (1 mark) lQ lQ lQ
Short Type I Answer (2 marks) - lQ
Short Type II Answer (3 marks) lQ lQ lQ
Long Answer (5 marks) lQ
O

, Value Based Questions (4 marks) I


I

• In 2015, only one question of 5 marks based on Biot-Savarts Law was asked in Delhi set.
• In 2016, in Delhi set, one question of 1 mark based on Moving coil Galvanometer and
10 ..rs one question of 3 marks based on Biot-Savart's Law were asked.
M

(. A iL ~' In 2017, in Delhi set, one question of 1 marks based on Induced Current, one question of
3 marks based on Magnetic Field were asked. In All India set, one question of 1 mark
based on Magnetic Field Lines, one question of 2 marks based on Magnetic Lorentz
Force and one question of 3 marks based on Magnetic Field were asked.
On the basis of above analysis, it can be said that from exam point of view Biot-Savarts Law,
Moving Coil Galvanometer, Magnetic Field and Magnetic Lorentz Force are most
important concepts of the chapter.
[TOPIC1] Magnetic Field Laws and
their Applications
1.1 Magnetic Field Permittivity and Permeability
The space in the surroundings of a magnet or a The relation between Ilo' Eo and c is

R
current carrying conductor in which its magnetic 1 2
--=c
influence can be experienced is called magnetic 1l0Eo
field. Its SI unit is tesla (T). where, c is velocity of light, Eo is permittivity of free
space and Ilo is magnetic permeability.
Oersted Experiment

SI
Oersted experiment Some Important Points Related to
demonstrated that

0-
the current carrying ~~K
Magnetic Field .
conductor carrying (i) Magnetic field at the
moving charges A- centre of a circular
produces magnetic current carrying
field around it.
When key Kis
\V
~ Conducting
wire
conductor/coil.
B=lloI
I
IT
Composs needle
closed, then 2r
deflection occurs in the compass needle and where, r is the radius of a circular loop.
vice-versa.
For N turns of coil, B = lloN1
2r
Magnetic Field due to a Current
H
(ii) Magnetic field at the centre of semi-circular
Element (Biot-Sovort's Low) current carrying conductor
According to this law,
B = 1101
the magnetic field due 4r
to small current carrying
I
O

element dl at any nearby


point P is given by
dB = h. Idlsin9
41t r-
or dB =h. Idl r (ill) Magnetic field at the centre of an arc of
M

41t Irl2 X circular current carrying conductor which


subtends an angle 9 at the centre
and direction is given by Current carrying conductor
Ampere's swimming B=hJ9
rule or right hand thumb rule. 41t r

where, h= 1O-7T-m/A=1O-7H/m
41t

and Ilo = permeability of free space or vacuum and


r = distance of point P from current carrying
element.
124 o ehopterwise eBSE Solved Papers PHYSICS

(iv) Magnetic field at any point lies on the axis (a) At any point inside the solenoid,
of circular current carrying conductor B = 1l0nI
2 where, n = number of turns per unit
B = 1l0Ia
2 (r2 + a2)3/2 length.

---"r + a
(b) At points near the ends of air closed
rx-, 2 2
, solenoid,

R
a'
,, ------_ P
<,
1
B = -llonI
2
(ii) Magnetic field due to a toroidal sole~
M

SI
~,'(ffflA4

~15 (/ ::s
(v) Magnetic field due to straight current
~//-"V
carrying conductor at any point P at a
distance r from the wire is given by
~ ::f
l--~-------
Long straight wire
P
v1----r~
IT
110 21 1
B=-·- => Boc- (a) Inside the toroidal solenoid, B = 11onI
41t r r N
Here, n =-
21tr

1.2 Ampere's Circuital Law and N = total number of turns


(b) In the open space, interior or exterior of
H
The line integral of the toroidal solenoid, B = 0
magnetic field B around I
any closed path in vacuum B
is equal to 110 times of the
total current I threading
PREVIOUS YEARS'
O

through the closed circuit t


i.e. loop. Mathematically, Magnetic EXAMINATION QUESTIONS
f B· dl= 1101 I
field lines
TOPIC 1
o 1 Mark Questions
M

Important Points
Related to Magnetic Field 1. A bar magnet is moved in the direction
• Magnitude of magnetic field of a straight wire indicated by the arrow between two coils
1 PQ and CD. Predict the direction of the
using Ampere's law B = 110

(i) Magnetic field due to a straight solenoid


21tT

PCJQ Nc==JS
---+ CCJO
induced current in each coil. All India 2017

i):-~:-:[-]-j A

2. Draw the magnetic field lines due to a


current carrying loop. Delhi 2013C
A
CHAPTER 4 : Moving Chorges and Magnetism 125

o 2 Marks Questions 10. An element M = Sxl y

is placed at the p
3. Two identical
origin (as shown in
circular loops P
figure) and carries a
and Q, each of
current I = 2 A.
radius rand
Find out the .0
carrying equal ~
magnetic field at a
currents are kept /tJ
point P on the I
in the parallel planes having a common I

R
Y -axis at a distance Zl
axis passing through O. The direction of
of 1.0 m due to the
current in P is clockwise and in Q is
element tu = w em. Also, give the
anti-clockwise as seen from 0 which is
direction of the field produced. Delhi 2009C
equidistant from the loops P and Q.
Find the magnitude of the net magnetic
o 3 Marks Questions

SI
field at O. Delhi 2D12
4. A long solenoid of length L having 11. (i) State Biot - Savart's law and express
N turns carries a current I. Deduce the this law in the vector form.
expression for the magnetic field in the (ii) Two identical circular coils, P and Q
interior of the solenoid. All India 2011C each of radius R, carrying currents
1 A and 13 A respectively, are placed
5. Obtain with the help of a necessary
concentrically and perpendicular to
diagram, the expression for the magnetic
IT
each other lying in the XY and YZ
field in the interior of a toroid carrying
planes. Find the magnitude and
current. HOTS; All India 2011C direction of the net magnetic field at
6. A straight wire of length L is bent into a the centre of the coils. All India 2017
semi-circular loop. Use Biot-Savart's law 12. Two identical loops P and Q each of radius
to deduce an expression for the magnetic
5 cm are lying in perpendicular planes
H
field at its centre due to the current I
such that they have a common centre as
passing through it. Delhi 2011C
shown in the figure. Find the magnitude
7. State Ampere's circuital law. Show through and direction of the net magnetic field at
an example, how this law enables an easy the common centre of the two coils, if they
evaluation of the magnetic field when there
O

carry currents equal to 3A and 4A,


is a symmetry in the system? All India 2010 respectively.
Q
8. State Biot-Savart's law. A Z
current I flows in a
conductor placed dl
P •
M

perpendicular to the plane O'r---.-"!:"""- y


of the paper. Indicate the P
A r·/ •
direction of the magnetic X
field due to a small element Delhi 2017
dl at a point P situated at a distance r from
the element as shown in the figure. Delhi 2009 13. Use Biot-Savart's law to derive the
expression for the magnetic field on the
9. A wire of length L is bent round in the
axis of a current carrying circular loop of
form of a coil having N turns of same
radius R.
radius. If a steady current I flows through
it in clockwise direction, then find the Draw the magnetic field lines due to a
magnitude and direction of the magnetic circular wire carrying current (1).
Delhi 2016
field produced at its centre. Foreign 2009
126 [2J ehapterwise eSSE Solved Papers PHYSICS

14. (i) State Ampere's circuital law 17. A long straight wire of a circular
expressing it in the integral form. cross-section of radius a carries a steady
(ii) Two long co-axial insulated solenoids current I. The current is uniformly
81 and 8 2 of equal length are wound distributed across the cross-section. Apply
one over the other as shown in the Ampere's circuital law to calculate the
figure. A steady current 1 flows magnetic field at a point in the region for
through the inner solenoid 81 to the (i) r < a and (ii) r > a. Deihl 2010

R
other end B which is connected to the
outer solenoid 82 through which the
some current I flows in the opposite [ZJ 5 Marks Questions
direction so, as to come out at end A. 18. (i) Write using Biot-Savart law, the
If n1 and n2 are the number of turns expression for the magnetic field B

SI
per unit length, find the magnitude due to an element dl carrying
and direction of the net magnetic current I at a distance r from it in a
field at a point vector form.
(a) inside on the axis and Hence, derive the expression for the
(b) outside the combined system. magnetic field due to a current
carrying loop of radius R at a point P
and distance x from its centre along
the axis of the loop.
IT
(ii) Explain how Biot-Savart law enables
one to express the Ampere's circuital
law in the integral form, uiz,
fB.dl =~oI
where, I is the total current passing
through the surface. Oelhl 2015
H
Dllhl2014 19. (i) State Ampere's circuital law. Use
this law to obtain the expression for
15. (i) How is a toroid different from a
the magnetic field inside an air cored
solenoid?
toroid of average radius r, having n
(ii) Use Ampere's circuital law to obtain
O

turns per unit length and carrying a


the magnetic field inside a toroid.
steady current I.
All India 2014C
(ii) An observer to the left of a solenoid
16. Figure shows a long straight wire of a of N turns each of cross-section areas
circular cross-section of radius a carrying
A observes that a steady current I
steady current I.The current I is
M

uniformly distributed across this flows in the clockwise direction.


cross-section. Derive the expressions for Depict the magnetic field lines due to
the magnetic field in the region (i) r < a the solenoid specifying its polarity
and (ii) r > a. All India 2011C and show that it acts as a bar
magnet of magnetic moment

'--'w
I
I I
I
\
, m = NIA All India 2015
I I I
1- I I I A 1"---"
I I I ' I
' I I
I I
\ , I I I I I I

'....... _-_ .... .•.


~ •••••••••• ~~~~~~ ••••• ~.I I I I
t\ :\ I
\
I
\
I
\
N\ \
\
, \
, \
\
CHAPTER 4 : Moving Charges and Magnetism 127

20. Two very small identical circular (iii) How is the magnetic field inside a
loop (1) and (2) carrying equal current I are given solenoid made strong?
placed vertically (with respect to the plane . All Indio 2011
of the paper) with their geometrical axes 24. (i) State Ampere's circuital law.
perpendicular to each other as shown in
(ii) Use it to derive an expression for
the figure. Find the magnitude and

R
magnetic field inside along the axis
direction of the net magnetic field produced
of an air cored solenoid.
at the point O.

IG~2-- --- - -- ____~o


(iii) Sketch the magnetic field lines for a
finite solenoid. How are these field
lines different from the electric field

SI
I
lines from an electric dipole?
I
Foreign 2010
: x
I
I
I 25. (i) Using Biot-Savart's law, deduce an
(2)
expression for the magnetic field on
the axis of a circular current
I
carrying loop.
Delhl2D14 (ii) Draw the magnetic field lines due to
. a current carrying loop.
21. State Biot-Savart's law expressing it in the
IT
vector form. Use it to obtain the expression (iii) A straight wire carrying a current of
for the magnetic field at an axial point 12 A is bent into a semi-circular arc
of radius 2.0 em as shown in the
distance d from the centre of a circular coil
figure. What is the magnetic field B
of radius a carrying current 1. Also, find
at 0 due to
the ratio of the magnitudes of the magnetic
(a) straight segments,
H
field of this coil at the centre and at an
(b) the semi-circular arc?
axial point for which d = a.Ji Delhl2013C Foreign 2010
22. State Biot-Savart's law and give the
mathematical expression for it.
Use this law to derive the expression for
O

the magnetic field due to a circular coil


carrying current at a point along its axis.
26. (i) State Ampere's circuital law. Show
How does a circular loop carrying current through an example, how this law
behave as a magnet? Deihl 2011 enables an easy evaluation of this
M

23. (i) Using Ampere's circuital law, obtain magnetic field when there is a
the expression for the magnetic field symmetry in the system?
due to a long solenoid at a point inside (ii) What does a toroid consist of? Show
the solenoid on its axis. that for an ideal toroid of closely
(ii) In what respect, is a toroid.different wound turns, the magnetic field
from a solenoid? Draw and compare (a) inside the toroid is constant.
the pattern of the magnetic field lines (b) in the open space inside an
in the two cases. exterior to the toroid is zero.
All Indio 2010C
o Explanations The Bis the magnetic field at any point inside the
solenoid.
1. By applying Lenz's law, we can find out direction Con;idering the rectangular closed path abcda.
of current in the coil. On the right hand side coil,
Applying Ampere's circuital law over loop abcda.(1J
South pole is approaching towards the coil, so at
end C, South pole will be produced and on the f B· dl= 110x (Total current passing through
left hand side, North pole is moving away, so at loop abcda)
. end Q coil, South pole will be produced. (1)
J>.dl+ J:B.dl+ fcdB.dl+ J:B.dl=llo(~li)

R
2. Magnetic field lines due to a current carrying loop
are given by
where, !!.. = number of turns per unit length
L
ab = cd = I = length of rectangle.
J: Ed! cos 0° + J: Bd! cos 90° + 0

SI
+ 1: Bd! cos 90°=110 (~)li

s rn B J: d!=llo (~)li => Bl=llo (~)li

3. To calculate net magnetic field at point 0, first of


all, calculate the magnetic field at point 0 due to => B=llo (~}
both coils with direction. By vector addition of
M B=llo~ m
IT
these two magnetic fields, net magnetic field can
where, n = number of turns per unit length.
be obtained.
This is required expression for magnetic field
Magnetic field at 0 due to two rings will be in
inside the long current carrying solenoid.
same direction (Q~ P, along the axis) and of
equal magnitude. (1/2) 5. Toroid is an endless solenoid to calculate the
magnetic field in the interior of toroid, Ampere's
B = Bl + B2 but B2 = Bl
circuital law can be obtained.
H
B = 281 = 2 [ 110 1r2 ] Toroid is a hollow circular ring on which a large
2 (r2 + r2)3/2 (1/2) number of insulated turns of a metallic wire are
2 2 closely wound.
B - 110Ir _ 110Ir
- (2r2)3/2 - i!2r3 The direction of the magnetic field at a point is
(112)
given by tangent to the magnetic field line at that
O

point. (1)
(1/2) B
/---

,I (J;>{\
......•..

4. Figure shows the longitudinal sectional view of


long current carrying solenoid. The current comes
out of the plane of paper at points marked.
\\V)
M

oQ
I+-- I ----+l
dj-----------"jc
I t
" ••.....
-- ./ I

---/1i.~
..:::
...~..·~f·.~... ~.• :::••. ~•••~•• ~•.• g'. ....(i)
a L. -----. b
as fB.dl = 110I x Number ofturns ....(ii)
~B -----------/

---~~mmmm~~1 If n be the number of turns/Unit length, then total


number of turns = n x 2ltr

So, fS dl = 110n x 2ltrl ....(ill)


CHAPTER 4 : Moving Charges ond Magnetism 129

From Eqs. (i) and (ill), we get 7. As, Ampere's circuital law states that the line
B 21tT = ~o n21tTI ~ B= ~o nl integral of magnetic field B around any closed
Q. loop is equal to ~o times the total current
threading through the loop. (1)

i.e. fB.dl=~oI

I I

R
Bounda~ Surtace
Applying Ampere's circuital law over loop,
To explain the Ampere's circuital law consider an
we have

SI
infinitely long conductor wire carrying a steady
f B· dl = ~o x Current passing through the loop (1)

6. When a straight wire is bent into semi-circular


loop, then there are two parts which can produce
the magnetic field at the centre, one is circular
part and other is straight part due to which field
':~:~:-"-r:-::~:
--------r--------
::'(tMg,mlal to
circumference)
is zero.
I
I
IT In order to determine the magnetic field at point P
r/
__ -.L-----.-----.l..- __- I which is situated at a distance R from the centre
r C r of the circular loop around the conductor wire, B
Length L is bent into semi-circular loop. (magnetic field) is tangential to circumference of
the loop. (1)
Length of wire = Circumference
of semi - equal circular wire Now,fB.dl= fBdl=B21tR=~oI
H
~ L = 1tT
L ~ B = ~o I [From Ampere's circuital law)
T =- ... (i) 21tR
1t
The direction of magnetic field will be determined
Considering a small element dl on current loop.
by right hand rule.
The magnetic field dB due to small current element
ldl at centre C. Using Biot-Savart's law, we have 8. Biot-Savart's law This law states that the
O

magnetic field (dB) at point P due to small current


dB = &.. Idl sin 90° ['r:' . .1 r, :. e = 90°) element Idl of current carrying conductor is
41t ,2
.. (112)
(i) directly proportional to the Idl (current)
dB=&'. Idl element of the conductor.
41t ,2 dB oc Idl
M

:. Net magnetic field at C due to semi-circular loop, y


B =f ~o Idl
semicircle 41t ,2
p
B = ~o I: f dl
41t,2 semicircle (1/2)

B = ~o . .!...- L
41t ,2
But, r = ~
1t
B = ~o . ....!!::....- = ~o x IL X 1t2 ~ B = ~o I1t x
(L/1t)2 L2 4L
(ii) directly proportional to sin e dB oc sin e
41t 41t (1)

This is the required expression. where, e is the angle between dl and r.


130 o Chapterwise eSSE Solved Papers PHYSICS

(ill) inversely proportional to the square of the 10. Biot-Savart's law states that
distance of point P from the current element. dB = 110 ,Idl x r
dB<x~ 47t Irl2 (1)
~ rn
Here, Idl = 2x wi
Combining all the inequalities
dB <X Idl sin 9 = 110 ,Idl sin 9
Sx = wcm
,2 47t ,2 ill = illx
1= 2A,' =1 m
where, 110 = 10-7 T-m/A for free space.

R
47t dB = 110 • (2wi x j)
The direction of magnetic field can be obtained 47t (1)2 (112)
using right hand thumb rule.
In v~ctor form, . •
r=j
dB = 110 ,Idlx r Irl=1 m

SI
47t Ir 12 (1/2) dB = 110w k ~ IdBI = 110w
z 27t 27t
and direction along +Z-axis. (1/2)
Current
element 11. (i) Biot-Savart's Law This law deals with the
magnetic field induction at a point due to a
)4-~-dB
: p small current element (a part of any
conductor carrying current).
y___ Current
IT ~~ - element

x (112)

The direction of magnetic field will be


perpendicular to Y-axis along upward in the plane
of paper.
H
9. When a straight wire is bent in
the form of a circular coil of

o
-::- X
N turns, then the length of the
According to Biot-Savarr's law, the magnitude
wire is equal to circumference of
of magnetic field intensity (dB) at a point P
the coil multiplied by the
due to a current element is given by, (1/2)
number of turns. Let the radius
O

dB idlsin9 <X
of coil be r,
,2
As, the wire is bent round in the form of a coil
having N turns. This relation is called Biot-Savart's law.
:. N x circumference of the coil If conductor is placed in air or vacuum, then
= Length of the wire dB = 110 idlsin9
,2
M

(27t') x N = L 47t
L
,=--
27tN
... (i) where, ~
47t
is a proportionality constant and 110

Magnetic field at the centre due to N turns of a coil is the permeability of free space.
is given by 110 = 47t X 10-7 Tm/A or weber/ ampere-metre.
B= 110 (NI) ~ B= 110 (NI) [From Eq. (i)] Thus, in vector notation
2r 2L~) dB <X idl x r = 110 . idl x r
,3 47t,3
Of 1
<jfn B = 1107tN21 The above expression holds when the medium
L (1Ya) is vacuum. The magnitude of this field is
The direction of magnetic field is perpendicular to IdBI = 110 . Idlsin9
the plane of loop and entering into it. (112) 47t ,2 (112)
CHAPTER 4 : Moving Charges and Magnetism 131

(ii) Magnetic field due to circular loop Q,


- ~OiQ
B Q---
2T (1)
p
So, net magnetic field at the common centre of

r r
the loop is,

Magnetic field due to circular wire P, Ene, = ~B; + B~ = (~;:p (~;;Q+

R
Bp = ~o X 2reI 1 (along vertically upwards)
7
4re R - ~o ~ _ 4re X 10- 5- 2 10-5 T
--"zP+ZQ- X - rex
= ~OIl 2r zx 5xlO-2 (1)
2R
Magnetic field due to circular wire 0..
Resultant magnetic field makes an angle
BQ which is given by,
e
J01-'51[
with
t:'

SI
BQ = ~o X 21tl2
tancp= Bp = ip =}.
4re R
BQ iQ. 4 (1)
(along horizontal towards left)
= ~oI2 13. Let us consider a circular loop of radius a with
2R (112) centre C. Let the plane of the coil be
Net magnetic field at the common centre of perpendicular to the plane of the paper and
the two coils, current I be flowing in the direction as shown in
the figure. Suppose P is any point on the axis at a
B=~ direction r from the centre.
IT Lm Q
~ B= (~)\(~r ""'" ~ (2 + a2 dB
.•.••••........

B= (~)\II2+Ii)
I

~ B=~O~II2+Ii
H
2R dB cos o
7 dB
B = 4re X 10- ~(l)2 + (.,[3)2 '------" Q'
2xR
7
B = 4re X 10- Tesla Now, consider a current element Idl on top (L )
O

R where current comes out of paper normally,


Resultant magnetic field makes an angle a whereas at bottom (M) enters into the plane of
with direction of BQ, which is given by paper normally.
tana= Bp =~ LP .l Idl
BQ .,[3 Also, MP .lIdl
~ a = 30°
M

(1/2) LP = MP = ~r2 + a2
12. Magnetic field due to circular loop P, The magnetic field at point P due to current
s, = Iloip element Idl. According to Biot-Savart's law,
2r dB = ~o . Idl sin 90°
Q
4re (r2 + a2)
where, a = radius of circular loop,
r = distance of point P from centre along the axis.
p
The direction of dB is perpendicular to LP and
along PO.. where PQ.l LP. Similarly, the same
magnitude of magnetic field is obtained due to
current element Idl at the bottom and direction is
along PQ', where PQ' .l MP.
132 o ehopterwise eBSE Solved Papers PHYSICS

Now, resolving dB due to current element at Land 16. In these types of questions, first of all we have
M dB cos cp components balance each other and net to calculate the current per unit area of
magnetic field is given by cross-section, so that we can calculate the
current in each loop, then only we can find the
B = fdB sin cP = f Ilo (~). a
41t r2 + a2 ~r2 + a2 magnetic field. The current is distributed
uniformly across the cross-section of radius a.

[
':In MeL, sin cP = a
~r2 + a2
1

R
= Ilo fa .!dl = Ilo fa (21tQ)
41t (r2 + a2)3/2 j 41t (r2 + a2)3/2
2
or B= llofa
J 2(r2 + a2)3/2

SI
_ lloN fa2
For N turns, B - 2 2 3/2 Tesla.
2(r +a ) (2)

The diagram of magnetic field lines due to a circular


wire carrying current I is.
Loop
(r < a)
.. I
:. Current passes per urut cross-section = --
1ta2
IT :. Current passes through the cross-section of
radius r is
2
(1) r = (~ x 1tr2) = 1r
2
... (i)
_ 1tQ2 a (112)
14. (i) Ampere's circuital law states that the line
integral of magnetic field (B) around any closed (i) cCder a loop of radius r whose centre lies
path in vacuum is Ilo times the net current (1) at the axis of wire where, r < a as shown in
H
threading the area enclosed by the curve. figure inside the wire.
Mathematically, f B·dl = Ilo I Applying Ampere's circuital law,
fB.dl=llo1'
Ampere's law is applicable only for an Amperian (1/2)
loop as the Gauss's law is used for Gaussian
surface in electrostatics. (1) f Bdl cosO° = Ilo ( ~2) [From Eq. (i)]
O

(il) According to Ampere's circuital .law, the net


magnetic field is given by B = Ilo n i (1) 1r2
B.!dl=llo -
(a) The net magnetic field is given by
t a2
2
Bnet = B2 -BI •"...
B x 21tr = _ _0_Ir ~ " _0Ir
B = _•... _
[':/2=11=1] ." a2 21ta2 (1/2)
=IlOn/2-llonl11
M

=llo1(n2-nJl ~ B cc r
The direction is from B to A. (1) . (ii) Considering a loop of radius r whose centre
(b) As the magnetic field due to SI is confined lies at the axis of wire and (r > a) as shown in
solely inside SI as the solenoids are outer dotted line.
assumed to be very long. So, there is no :. Current I threads the loops.
magnetic field outside SI due to current in Applying Ampere's circuital law,
SI' similarly there is no field outside S2'
fB.dl=llo1
:. Bnet = 0 (1) (1/2)

15. (i) A toroid can be viewed as a solenoid which has f Bdlcosoo=llo1 ~ Bf dl=llo1
been bent into circular shape to close on itselfrn
(il) Refer to Ans. 14 (il). (1) B x 21tr =llo1 ~ B = llo1
21tr (1)
CHAPTER 4 : Moving Charges and Magnetism 133

Boc~ According to Biot-Savart's law,


r dB= llo .Idlsin90°.
Thus, the field B is proportional to r as we move 4n (r2 + a2)
from the axis of cylinder towards its surface and
where, a = radius of circular loop
then decreases as ~ .
r r = distance of point P from centre along the axis.

17. Refer to Ans. 16. (3)


The direction of dB is perpendicular to LP and
along po, where PQ 1. LP. Similarly, the same
18. (i) According to Biot-Savart's law, conductor is

R
magnitude of magnetic field is obtained due to
placed in air or vacuum, current element Idl at bottom and direction is
Current
along PQ', where PQ 1. MP.
element
Now, resolving dB due to current element at L and
6
Id I H--, -'----+lB MdBcos$ components balance each other and net
, magnetic field is given by

SI
~ B = fdBSin$= f!:Q.( /dL 2)' a
4n r + a ~r2 + 12

then dB = llo idlsin6 ..:In MC/.:, a 1


4n r2
[ sm$=-;===
In vector form, Biot-Savart's law can be written as, ~r2 + a2
dB oc Idl x r B = llo . Ia I.dl
r3 4n (r2 + a2)3/2 j
IT
_lloIdlxr 2
- 4n-r-3- B = llo . Ia (2na) = lloIa
4n (r2 + a2)3/2 2 (r + a2)3/2
2
Let us consider a circular loop of radius a with 2
centre C. Let, the plane of the coil be For N turns, B= lloNla
perpendicular to the plane of the paper and 2(r2 + a2)3/2
current I be flowing in the direction shown in the The direction is along the axis and away from the
H
figure. loop. (2'12)
(ii) When current in the coil is in anti-clockwise
---..,.Q direction.
y
dB
O

dB cos e
dB
~_---2llQ'
M

Suppose P is any point on the axis at a direction r


from the centre.
Now, consider a current element Idlon top L,
where current comes out of paper normally
whereas at bottom (M) enters into the plane
paper normally.
.. LP 1. dl
Also, MP 1. dl
.. MP = LP = ~r2 + a2
The magnetic field at P due to current element ldl,
134 [2] Chopterwise C8SE Solved Popers PHYSICS

Consider any arbitrary dosed path perpendicular


to the plane of paper around a long straight
conductor XY carrying current from X to Y, lying
in the plane of paper. loop 1
Let, the closed path be made of large number of loop 2
small elements, where -O(Jh--1~ ...""-Ioop 3
AB = dll' BC = dl2, CD = dl}
Let del' de2, de}, be the angles subtended by the
various elements at point 0 through which

R
For loop 1,
conductor is passing. Then
de I + de 2 + de J + ... = 21t According to Ampere's circuital law, we have,
Suppose these small elements AB, BC, CD, ... are fB. dl = Ilo(total current)
small circular arcs ofradii '1' '2' ,} ,... respectively.
Total current for loop 1 is zero because no
dll di2 dl}

SI
Then, del = -, de2 = -, de} = - current is passing through this loop.
1j '2 ,} So, for loop 1, fB. 81 = 0
If BI, B2, B} are the magnetic field induction at a For loop 3,
point along the small elements dl" dl2 ,dl3 •••••• According to Ampere's circuital law, we have,
then from Biot-Savart's law we know that for the fB. dl = Ilo (total current)
conductor of infinite length, magnetic field is
given by Total current for loop 3 is zero because net
BI = Ilo 21 ; B2 = Ilo 21; B} = Ilo 21 ... current coming out of this loop is equal to the
41t '1 41t '2 41t ,}
net current going inside the loop.
IT For loop 2,
In case of each elements, the magnetic field
induction B and current element vector dl are in The total current flowing through the toroid is
the same direction. Line integral of B around NI. Where, N is the total number of turns.
closed path is fB.dl=llo(Nl) ...(i)
fB.dl=BI·~ +B2·dl2+B}·dl}+ ...
Now, Band dlare in the same direction.
= BI (d4) + B2(d~) + B}(d4) + ... fB.dl=Bfdl=> fB.dl=B(21tT) .... (ii)
H
= Ilo 21 d4 + Ilo 21 d~ + Ilo 21 d13+...
On comparing Eqs. (i) and (ii), we get
41t '1 41t T2 41t ,}
=> B = lloN1
= llo21[d4 + d~ + d4 + ... J 21tT
41t TI T2 T} Number of turns per unit length is given by
N
O

= llo21 [del + de2 + de} + ... J= Ilo 21 x 21t = llo1 n=-


41t 41t 21tT
.. B=lloN1
=> fB. dl = Ilol, which is an expression of
This is the expression for magnetic field inside
Ampere's circuital law. (2'12) air-cored toroid. (1'12)

19. (i) Ampere's circuital law states that the line (ii) Since, it is given that the current flows in the
M

integral of magnetic induction B around a clockwise direction for an observer on the left
closed path in vacuum is equal to Ilo times the side of the solenoid. It means that the left face
total current I passing through the surface, i.e. of the solenoid acts as South pole and right
fB. dl = Ilol. (1) face acts as North pole. Inside a bar, the
A toroid is a hollow circular ring on which a magnetic field lines are directed from South to
large number of turns of a wire are closely North. (1'12)
wound. Therefore, the magnetic field lines are directed
Now, consider an air-cored toroid with centre from left to right in the solenoid. (1)
o and in order to determine the magnetic field Magnetic moment of a single current carrying
inside the toroid, we consider three amperian loop is given by, m' = lA .
loops So, magnetic moment of the whole solenoid is
(loop 1, loop 2 and loop 3). given by m = Nm' = N(lA) . (1)
CHAPTER 4 : Moving Charges and Magnetism 135

20. The magnetic field at a point due to a circular 22. For Blot-Savart/s law Refer to Ans. 8. (2)
loop is given by For the magnetic field due to a circular coil
2
B = ~O. 2n:la carrying current at a point along its axis. Refer to
4n: (a2 + r2)3/2 Ans.13. (1)
(1)
where, I = current through the loop As current carrying loop has the magnetic held
lines around it which exists a force on a moving
" = radius of the loop
charge. Thus, it behaves as a magnet with two
and r = distance of 0 from the centre of the loop.
mutually opposite poles. (1)
Since I, a and r = x are the same for both the

R
loops, the magnitude of B will be the same and is
given by (1)
2
Bl = B2 = !:.Q... 2n:la
4n: (a2 + x2)3/2
The direction of magnetic field due to loop (1 ) The anti-clockwise flow of current behaves like a

SI
will be away from 0 and that of the magnetic North pole, whereas clockwise flow as South pole.
field due to loop (2) will be towards 0 as shown. Hence, loop behaves as a magnet. (1)

The direction of the net magnetic field will be as 23. (i) Refer to Ans. 4. (2)

shown below: (ii) Solenoid is a hollow circular ring having large


(1)
Bnet : number of turns of insulated copper wire on it.
Therefore, we can assume that toroid is a bent
solenoid to close on itself.
_________________________
0) _: 0
~ The magnetic fields due to solenoid and toroid
is given in figures below: (1)
90°
IT
x

(2)
H
(1)

The magnitude of the net magnetic field is given


by
Field due to solenoid
_ ~ _ ~o 2.J2itIa2
O

Bnet -" B1 + B2 ~ Bnet - 2 2 3 2


4n: (a + x ) 1 (1) ,/,,- - ~ -- ---- ------ ---,
21. For the statement of Biot-Savart's law Refer 3,,' ,,
,,
to Ans. 8. (1) ,,
I
, ,
I \
For field at axial point Refer to AIlS. 18 (i). (1) I
I \
\
I I

= d.
M

I I
In this answer, put r I
I I
I

I I
\ I
Magnetic field induction at the centre of the ,
, I
I

, I

circular coil carrying current is ,


,, , I

, ,,
B =!:.Q... 2n:I
2
4n: a "',,---------------,,/
2 Field due to toroid
_ ~o 2n;a I
Bl - 4n:' (a2 + d2)3/2 Magnetic field inside the solenoid is a uniform,
B1 _ a2xa a3
strong and along its axis also field lines are all
[':d = a.J3] most parallel while inside the toroid field line
B2 (a2 + d2)3/2 (a2 + d2)3/2
makes closed path.
Bt _ a3 _ a3 _ a3 (iii) The magnetic field in the solenoid can be
B2 - (a2 + 3a2)3/2 -(4a2)3i2" - 8a3
(3) increased by inserting a soft iron core inside it. (2)
136 o Chapterwise CBSE Solved Papers PHYSICS

24. (i) For statement of Ampere's circuital Iaw _ 1 (~oI) _---


~oI
Refer to Ans. 7. (1)
-- --
2 2r 4r
(ii) Refer to Ans. 4. (2)
(iii) Magnetic field lines due to a finite solenoid ~ B = ~oI = (41t x 10-7) x 12 = 61t X 10-5 T
has been shown as below: 4r 4x2xlO-2 (1'I2J
Q
26. (i) For statement of Ampere's circuital law
Refer to Ans. 7.

R
For the evaluation of magnetic field for a
symmetrical system, we can consider the
example of a current carrying solenoid. Now,
refer to Ans. 4. (1V.)
(ii) A solenoid bent into the form of closed loop is
called toroid. The magnetic field B has a

SI
constant magnitude everywhere inside the
toroid. (112)
Q.
IT
(2) I I
All the magnetic field lines are necessarily closed
loops, whereas electric lines of force are not. (a) Let magnetic field inside the toroid is B along
H
the considered loop (1) as shown in figure.
25. (i) Refer to Ans. 8 and 18. (1)
(ii) Magnetic field lines due to a current carrying ///-------------""',9 B
loop is given as below: (2) 3r , ,,
r ,
I ,
I ,
I ,
O

I I
I I
I I
I I
I I
I I
I I
, I
, I
,
,, ,,
I

,, ,,
-, <, --------_ ,;/'
F
M

Applying Ampere's circuital law,


(ill) Magnetic field due to straight part
B - f~O Idlx r !
iloopl
B·dl=~o (NI)
41t r3 (112)
For point 0, dl and r for each element of the Since, toroid of N turns, threads the loop 1,
straight segments AB and DE are parallel. N times, each carrying current I inside the loop.
Therefore, dl x r = O. Hence, magnetic field due Therefore, total current threading the loop 1 is
to straight segments is zero. NI.
Magnetic field at the centre due to circular ~! B·dl=~oNI ~ B! dl=~oNI
iloop 1 iloop
point
Magnetic field at the centre of circular coil B x 21tr = ~o NI
2 or B = ~oNI
[":Here, coil is half 1 21tr (1)
CHAPTER 4 : Moving Charges and Magnetism 137

(b) Magnetic field inside the open space Here, each turn of toroid threads the loop two
interior the toroid. Let the loop (2) is shown times in opposite directions.
in figure experience magnetic field B. Therefore, net current threading the loop
No current threads the loop 2 which lie in the = NI -Nl = 0
open space inside the toroid. :. By Ampere's circuital law,
:. Ampere's circuital law
1, B.dl=l.I.o(Nl-NI)=O
1, B·dl=l.I.o(O)=O:::) B=O "loop 3
~~2 rn :::) B=O
Magnetic field in the open space exterior

R
of toroid Let us consider a coplanar loop (3) Thus, there is no magnetic field in the open space
in the open space of exterior of toroid. interior and exterior of toroid. (1)

[TOPIC 2] Lorentz Force and Cyclotron

SI
2.1 Force on moving charge in a Magnetic Force on a Charged Particle
uniform Magnetic and • Work done by magnetic Lorentz force on charge
particle is zero as F 1- v, hence F is perpendicular
Electric Field (Lorentz Force)
to displacement.
Force experienced by a single charged particle q
• Magnetic force cannot increase the kinetic
moving with speed v in a uniform magnetic field
IT
energy of charge particle.
at an angle a with it is known as magnetic
• The trajectory/path traversed by the charged
Lorentz force.
particle is a
F = q (v x B) [vector form]
(i) straight line when angle between vand B is
Magnitude of F = qvB sin a and direction of force 0° or 180°.
is given by right hand palm rule or Fleming's left
H
(ii) circle when angle between v and B is 90°.
hand rule.
(iii) helix when angle between v x B is an acute
51 Unit of Magnetic Field angle.
Magnetic lorentz force, F = qvB sin a • When charge particle enter in a magnetic field
perpendicularly, then
O

=> B= __ F_
qv sin a . mv' '" mv
(z) -- = qvB (II) radius, r = -
r qB
If F = IN, q = I C, v = 1 ms-1
(iii)
III T = --21tm (.zv ) f = --qB
sin a = 1= sin 90 ° => a = 90 ° qB 21tm
M

q2B2 2
.. SI unit of B = • 1N I r
(v) KE=--
(lC)(lms-) 2m
= 1 NA - lm- I = 1T • When angle between v and B is a, then
SI unit of magnetic field, B = 1tesla (T).
(z') Ra diIUS 0
f h e ulCa1 pat h ,r = mv sin a = --mv.l
qB qB
Right Hand Palm Rule where, v.l = vsina= perpendicular component
If all four fingers of right hand together points in of velocity.
the direction of magnetic field and thumb points
the direction of motion of positive charge, then ~Helixes
the palm of right hand faces in the direction of
force.
138 o ehopterwise eBSE Solved Papers PHYSICS

(ii) Time period, • KEmax of charge particle accelerated by


_ 21tr _ 21tr mvsinB _ 21tm q2B2R2
T ----_. --- cyclotron is KE = ---
v.L vsine qB qB max 2m

(iii) Frequency, f = qB
where, R = radius of circular track of charged
21tm particle.
(i ) P' h 21tm v cosa. 21tmvll • Cyclotron cannot accelerate electron and
IV itc = =--
electrically neutral particles.
qB qB

R
where, vII = vcose = parallel component of
velocity .
• Net force on a charged particle when both
electric and magnetic fields are present. PREVIOUS YEARS'

SI
F=q [E+(vxB)]
• If a charged particle q accelerated by potential
.EXAMINATION QUESTIONS
difference V and speed changes from 0 to v, TOPIC 2
then,
Work done = Change in KE 01 Mark Questions
qV = -1 mv 2 => v = ~2qv
-- 1. Write the expression in a vector form for
2 m
IT the Lorentz magnetic force F due to a
charge moving with velocity v in a
2.2 Cyclotron magnetic field B. What is the direction of
A compact device used to accelerate the positively the. magnetic force? Delhi 2014
charged ion or particles (like protons, deuterons, 2. A narrow beam of protons and deuterons,
a-particles) up to very high speed. each having the same momentum, enters
H
. a region of uniform magnetic field
N directed perpendicular to their direction
of momentum. What would be the ratio of
the radii of the circular path described by
them? Foreign 2011
O

3. Two particles A and B of masses m and


2m have charges q and 2q respectively.
They are moving with velocities VI and V2
respectively in the same direction, enters
M

the same magnetic field B acting normally


to their direction of motion. If the two
forces FA and FB acting on them are in the
ratio of 1 : 2, find the ratio of their
velocities. Delhi 2D11C
4. A beam of a-particles projected along
Cyclotron
+ X-axis, experiences a force due to a
Based on the principle of magnetic resonance, a magnetic field along the + Y -axis. What is
charge particle can be accelerated to high speed by the direction of the magnetic field?
passing it again and again through small region of
All Indio 2010, 2008
oscillating electrical field by making the use of
strong normal magnetic field.
CHAPTER 4 : Moving Charges and Magnetism 139

5. Use the expression F = q (v x B) to define 13. A deuteron and a proton moving with the
the S1 unit of magnetic field. All India 2010C same speed enter the same magnetic field
region at right angles to the direction of
6. An electron does not suffer any deflection
the field. Show the trajectories followed
while passing through a region of a
by the two particles in the magnetic field.
uniform magnetic field. What is the
direction of the magnetic field? All India 2009
Find the ratio of the radii of the circular
paths which the two particles may

o 2 Marks describe. Foreign 2010

R
Questions B
14. A charge q moving
7. Find the condition under which the charged along the X-axis with a
Z
particles moving with different speeds in
velocity v is subjected
the presence of electric and magnetic field
to a 'uniform magnetic
vectors can be used to select charged

SI
field B acting along the o'}------_y
partie' ,of a particular speed. All India 2017
Z-axis as it crosses the ~
8. Define one tesla using the expression for origin O.
the magnetic force acting on a particle of x
(i) Trace trajectory
charge q moving with velocity u in a
(ii) Does the charged particle gain
magnetic field B. Foreign 2014
kinetic energy as it enters the
9. State the underlying principle of a magnetic field? Justify your answer.
cyclotron. Write briefly how this machine HOTS; Delhi 2009
IT
is used to accelerate charged particles to
15. A point charge is y
high energies. Delhi 2014
moving with a
10. A particle of charge q and mass m is constant velocity x x x x x
B
moving with velocity v. It is subjected to a perpendicular to a x x x x x
uniform magnetic field B directed uniform magnetic x X
-+q
Vx x )(
perpendicular to its velocity. Show that it field as shown in the
H
x x x x x
describes a circular path. Write the figure. What should x x x x x
expression for its radius. Foreign 2012 be magnitude and ~-----------x
11. Write the expression for Lorentz magnetic direction of the
force on a particle of charge q moving with electric field, so that the particle moves
O

velocity v in a magnetic field B. Show that undeviated along the same path? Foreign 2009
no work is done by this force on the
16. A cyclotron when being used to accelerate
charged particle. All India 2011
positive ions? (Mass = 6.7 x 1O-27kg,
12. An electron and a proton moving with the charge = 3.2 x 1O-19q has a magnetic field
same speed enter the same magnetic field
of(1t/2)T. What must be the value of the
M

region at right angles to the direction of


the field. Show the trajectory followed by frequency of the applied alternating
the two particles in the magnetic field. electric field to be used in it? All India 2009
Find the ratio of the radii of the circular
paths which the particles may describe.
Foreign 2010
o 3 Marks Questions
x x x x 17. (i) Write the expression for the force
F acting on a particle of mass m and
x x x charge q moving with velocity u in a
magnetic field B. Under what
ee- x x x x
conditions will it move in
(a) a circular path and
x x x x
B
(b) a helical path?
140 o Chapterwise CBSE Solved Papers PHYSICS

(ii) Show that the kinetic energy of the 23. (i) Write an expression for the force
particle moving in magnetic field experienced by a charge q moving
remains constant. Deihl 2017 with a velocity v in a magnetic
18. (i) Write the expression for the field B. Use this expression to define
magnetic force acting on a charged the unit of magnetic field.
particle moving with velocity v in the (ii) Obtain an expression for the force
presence of magnetic field B. experienced by a current carrying
(ii) A neutron, an electron and an alpha wire in a magnetic field. All India 2D09C

R
particle moving with equal velocities,
enter a uniform magnetic field going o 5 Marks Questions
into the plane of the paper as shown
in the figure. Trace their paths in 24. (i) Deduce an expression for the
the field and justify your answer. frequency of revolution of a charged

SI
particle in a magnetic field and show
x x x x x X
that it is independent of velocity or
<X-
energy of the particle.
X X X X X x
n_ (ii) Draw a schematic sketch of a
x x x x x x cyclotron. Explain the essential
e e--------+ details of its construction how it is
x x x x x x used to accelerate the charged
CBSE Delhi 2016 particles? Delhi 2014
IT
19. A uniform magnetic field B is set up along 25. (i) Draw a schematic sketch of a
the positive X-axis. A particle of charge q cyclotron. Explain clearly the role of
and mass m moving with a velocity venters crossed electric and magnetic field in
the field at the origin in XY-plane such that accelerating the charge. Hence, derive
it has velocity components both along and the expression for the kinetic energy
acquired by the particles.
perpendicular to the magnetic field B.
H
Trace, giving reason, the trajectory followed (ii) An a-particle and a proton are
by the particle. Find out the expression for released from the centre of the
the distance moved by the particle along cyclotron and made to accelerate.
the magnetic field in one rotation. (a) Can both be accelerated at the
same cyclotron frequency? Give
O

All India 2011


reason to justify your answer.
20. Draw a schematic sketch of the cyclotron. (b) When they are accelerated in
State its working principle. Show that the turn, which of the two will have
cyclotron frequency is independent of the higher velocity at the exit slit of
velocity of the charged particle. Delhi 2D11C the does? Delhi 2012
M

21. Explain the principle and working of a 26. Write the expression for the force F, acting
cyclotron with the help of a schematic on a charged particle of charge q moving
diagram. Write the expression for with a velocity v in the presence of both
cyclotron frequency. Delhi 2009 electric field E and magnetic field B.
Obtain the condition under which the
22. Show that the frequency of revolution of a
particle moves undeflected through the
charged particle (In the XY-plane), in a
fields. All India 2012C
uniform magnetic field B (B = Bok), is
independent of its speed. Which practical
27. With the help of a labelled diagram, state
the underlying principle of a cyclotron.
machine makes the use of this fact? What
Explain clearly how it works to accelerate
is the frequency of the alternating electric
the charged particles? Delhi 2011
field used in this machine? Delhi 2009C
CHAPTER 4 : Moving Charges and Magnetism 141

Show that cyclotron frequency is As they have same momentum and charge
independent of energy of the particle. Is moving in a small magnetic field. (1)
there an upper limit on the energy Hence, rp :rd = 1 : 1
acquired by the particle? Give reason. 3. Ratio of forces acting on the two particles
28. (i) Draw a schematic sketch of a FA = qVI B sin 90° 1
cyclotron, explain its working FB (24) v2 Bsin 90° 2
principle and deduce the expression [BI = B2, same magnetic field)
for the kinetic energy of the ions

R
According to the question,
accelerated.
(ii) Two long and parallel straight wires ~ ~ =1 ~ VI : v2 = 1 : 1
v2 (1)
carrying currents of 2A and 5 A in the
opposite directions are separated by 4.. : Velocity of a:particles
a distance of 1 cm. Find the nature

SI
v = vi [Projected along X-axis)
and magnitude of the magnetic force
.: Magnetic force on a-particles
between them. Foreign 2011
Fm = q (v x B) = q (vi x B)
29. Draw a schematic sketch of a cyclotron.
Fm = Fm j [Oriented along Y-axis)
State its working principle. Describe
briefly, how it is used to accelerate ~ Fm j = q (vi x B)~ B = - Bk = B(-k)
charged particles? Show that the period of The direction of magnetic field must be along
a revolution of an ion is independent of its Z-direction. (1)

speed or radius of the orbit. Write two 5. Given, F = q (v x B)~ F = qvB sin e
IT
important uses of a cyclotron. All India 2011C where, e is the angle between v and B.
30. Draw a schematic sketch of cyclotron. ~ B= __F_
Explain briefly how it works and how it is qv sin e
used to accelerate the charge particles? i.e. ifq =1 C v=1 ms-',e= 90°, thenB=F
(i) Show the time period of ions in a The magnetic field at any point is given by
H
cyclotron is independent of both the B=_F_= IN =IN/A-m=l T
speed and radius of circular path. qv sin e [(1 C) (1ms ') sin 90°) (1)

(ii) What is resonance condition? How is :. S1unit of magnetic field = 1T


it used to accelerate the charged Thus, the magnetic field induction at a point is said
particles? Foreign 2009; All India 2009 to be one tesla if a charge of one coulomb while
O

moving at right angle to a magnetic field with a

o Explanations velocity of 1 ms " experiences a force of 1 N at that


point.
1. The expression in vector form is given by 6. An electron does not suffer any deflection in the
F = q(v x B) (1/2) magnetic field, it means that the electron is
M

moving parallel or antiparallel to the magnetic


where, q is the charged particle.
field, i.e. along or opposite to the direction of
The direction of the magnetic force is in the magnetic field. (1)
direction of vx B, i.e. perpendicular to the plane
7. A diagram in which particle moves in magnetic
containing v and B. (112)
and electric field is shown below (1)
2. For the given momentum of charge particle, x x x B
radius of circular paths depends on charge and + + + +
magnetic field as
mv 1
r=- ~ roc- [.: qvB = m:] v
qB qB x

For constant momentum,


x
:. r proton : r deuteron = 1 : 1 = q deuteron : q proton
142 o Chapterwise CSSE Solved Papers PHYSICS

Forces on a charged particle are Every time, the particle moves from one dee to
F. = electric force = qE the other it comes under the influence of
Fm = magnetic force = Bqv electric field which ensures to increase the
energy of the particle as the sign of the electric
For a particle to go straight without any deflection;
field changed alternately.
E The increased energy increases the radius of
Fe = Fm ~ qE = Bqv ~ v = -B
the circular path, so the accelerated particle
In this way, particles having speed, v = -E are moves in a spiral path.
B Since, radius of trajectory

R
separated. (1)

8. One tesla is defined as the field which produces a


[as we know, m: = BqV]
force of I newton when a charge of I coulomb
vm
moves perpendicularly in the region of the r=-
qB
magnetic field at a velocity of! ms'" .

SI
rqB
F IN V=-
F=qvB ~ B=- ~ I T=----,- m
qv uc) (I ms") (2)
Hence, the kinetic energy of ions
9. In cyclotron, Lorentz force concept is utilised. I I r2q2B2 I r2q2B2
Force is acting perpendicular to both velocity and =-m;=-m--~KE =---
22m2 2m (1)
magnetic field.
10. A charge q projected B
Principle A charged particle can be accelerated
perpendicular to the x
to very high energies by making it pass through a
uniform magnetic field B x
moderate electric field a number of times. This
IT
with velocity v. The
can be done with the help of a perpendicular •v
perpendicular force,
magnetic field which throws the charged particle
F = qvB, acts like a
into a circular motion, the frequency of which
centripetal force
does not depend on the speed of the particle and
perpendicular to the
the radius of the circular orbit.
magnetic field. Then, the x
The combination of crossed electric and magnetic
H
path followed by charge is
fields is used to increase the energy of the charged
circular as shown in the figure. (1)
particle. Cyclotron uses the fact that the frequency
of revolution of the charged particle in a magnetic m; mv
qvB = -- or r = -
field is independent of its energy. r qB
Magnetic field out where, r = radius of the circular path followed by
O

of the paper Deflection plate charge projected perpendicular to a uniform


magnetic field. (1)

11. Lorentz force always acts along the direction


perpendicular to the direction of velocity of the
particle.
M

Magnetic Lorentz force,


Fm = q (v x B) (1J
F 1. v
~ Force is perpendicular to displacement made by
charge particle.
.. W = Fd cos 90 = 00

[:. Force F and displacement dare


Oscillator (1) perpendicular to each other]
Inside the dees the particle is shielded from ~ W=O
the electric field and magnetic field acts on the No work is done by magnetic Lorentz force on the
particle and makes it to go round in a circular charged particle. (1)
path.
CHAPTER 4 : Moving Charges and Magnetism 143

12. When a charged panicle enters iri the magnetic 14. When a force acts along the direction perpendicular
field at right angle, then the particle follows a to the direction'of the velocity of a particle, then no
circular path. work is done on the particle by this force.
x x x x (i) As the charged particle is moving perpendicularly
Circular to the magnetic field. So, it will perform circular
path motion in XY - plane.
x x Z
p-

R
x x x x
a--
~ectron }----- ...•.y
x
x Circular x x
q
/ I"'"
,".--. ""'\

SI
path I
\ ,
I

" ....._-.....--, ... '"


Radius of the circular path, r = mv X m
qB (ii) No, the charged particle does not gain any KE as
For same speed v, magnitude of charge and Lorentz force acting on it does not perform any
magnetic field work as Fm .1 v. (1)
roc m W = Fdcos90o= 0
!!. = m,
15. v=- vi
rp rrIp (1)
[': The particl~ is moving along X-direction]
IT
where, m, and. mp are masses of electron and
proton, respectrvely B=-Bk
m, <mp [.: The magnetic field is perpendicular to
(Proton is much heavier than electron) the plane of the paper directed inwards, i.e.
l -direction]
~ r,<rp
:. Force acting due to magnetic field, .
The curvature of path of electron is much more
H
than curvature of path of proton. (1) Fm = q (v x B) = q [- vi x (- B~]

13. Refer to Ans. 12. (1) Fm=-qvBJ [.: ixk=-1](1)


rd _ md
---, => Magnitude of Fm = IFm I = qvB
rp mp The direction is along Y-direction.
O

For the undeflected motion of particle,


md = 2rr1p
Force due to electric field = Force due to magnetic field
~ rd = 2rp qE= q(v x B)
or rd:rp=2:1 (1) .. E=vxB
l( l( l( l(
Magnitude of electric field, lEI = [v x B1 and direction
Deuteron of magnetic field will be perpendicular to both v and
M

B, i.e. along Y-axis. (1)


l( l(

16. .: Frequency of alternating electric field cyclotron is


given by f = qB
l( l( l( l(
2nm (1/2)
19
Here, q = 3.2 X 10- C m = 6.7 x 10-27 kg

and B= ~T
l( l( l( l( 2
f = (3.2 x 10-19) x (nI2)
NOTE Smaller the radius, greater the curvature and 2 x n x 6.7 x 10-27 (1)
vice-versa. This is why, because proton's path
has got greater curvature. f = 1.2 X 10 cycle/s7
(112)
144 o Chapterwise C8SE Solved Papers PHYSICS

17. (i) Force acting on the particle, F = Bqv For electron


In vector form, F =q(vx B) Force is given by F = - e(v x B)
where, B is uniform magnetic field and v is So, direction = - (i x - k) =}- j
velocity with particle which is moving. (e- describes a circle with clockwise motion)
From this equation, it is clear that direction of ® '~ ®
force is perpendicular to the plane containing I

both v and B . In other words, force acts ®


a..........-.:' 0 ®
perpendicular to both v and B. When velocity ® ® ®

R
becomes perpendicular to force, the path of the ® ® ®

"~'L
object becomes circular. (1)
n_--- •.-----
® ® ®
e---+--.,
® ~ ®
I

e ,tiJ ®
(2)

SI
19. y
v sin 9 B B-+

In this case, B is assumed to act perpendicular


Vol =V sin 9
to v. In case, B is not perpendicular to v, a v-+
component of v remains perpendicular to v. It t
creates circular path. The component of
v parallel to B will create linear path.
IT
Here, circular path is due to vcos 9 and linear
path is due to vsin9. Both when combined gives
helical path. (1)

(ii) Since, force always adjusts itself in a direction


The path of the charged particle will be helix. As,
which becomes perpendicular to velocity, so
the charge moves linearly in the direction of the
only direction of velocity is changes not the
magnetic field with velocity vcos9 and also
magnitude. Hence, the kinetic energy of the
H
describe the circular path due to velocity vsin9.
-particle always remains constant. (1)
Time taken by the charge to complete one circular
18. (i) When a charged particle (q) moves with
velocity (v) inside a uniform magnetic field B, rotation, T = 27tr (1)
vol
then force acting on it is
F=q(vxB) f= qVolB ... (i)
O

From right hand thumb rule, the force F is


perpendicular to the velocity (v) and magnetic and
mvi = qVolB
-- ... (ii)
r
field (B).
Hence, it changes its path continuously. (1) From Eqs. (i) and (ii), we get
(ii) According to question, magnetic force on a vim
--=qvolB
charge F particle is given by
M

r
F =q (vxB)
The direction of force on the charged particle is =} Volm = r =} T = 27tvolm =} T = 27tm
given by (v x B) with the sign of charged qB qB· Vol Bq (1)

particle, i.e. for a-particle, charge is positive Distance moved by the particle along the
and direction of v is + i and direction of B is -k. magnetic field in one rotation (pitch of the helix
So, direction of force is + (i x- k) , i.e. + j. path)
For a-particle = VII X T t: VII = V parallel)
It describes a circle with anti-clockwise
motion. = V cos9x 27tm
Bq
For neutron
It is a neutral particle so, it goes undeviated. p = 27tmvcos9 = 27tmvcos9
As F = q (v x B) = 0 qB qB (1)
CHAPTER 4 : Moving Charges and Magnetism 145

20. (i) Schematic diagram of cyclotron is shown as The charge d particle again accelerated towards Dl
below: as D2 acquires positive and d negative polarity.
Thus, the charge particle is brought again and
again in the small region of oscillating electrical
field by strong normal magnetic field.
The charged particle repeatedly passes through
oscillatirlg electrical field. It traversed on spiral
path and finally having radius of its circular path

R
becomes equal to the radius of dees and finally
comes out through window Wand strikes to the
D1
target. n)
22. For first part refer to Ans. 9. (1)
Cyclotron is that machine which uses the above
fact. n)

SI
The frequency of alternating electrical field is
Target
equal to the frequency of charge particle which is
B .
given by f = -q-.
(1) 2~ m
(ii) For Principle of Cyclotron Refer to Ans. 9. 23. (i) The required expression for the face is
(1) F = q (v x B) = q vB sin e
(ill) Radius of a charged particle,
Now, refer to Ans. 5. (1)
mv
r=- (ii) Consider the segment of a conductor given in
qB
IT the below figure.
~ Time period of charged particle
T = 21tr = 21tm
v qB
~ Frequency, f = qB =.!..
znm T
Let the number of electrons per unit volume of
H
From the above formula, it is clear that frequency of
charged particle does not depend on the velocity of the conductor is n, the drift speed of electron
charged particle. (1)
inside the conductor is vd and the magnetic
field is B.
21. Principle of Cyclotron The cyclotron works on Then, Lorentz magnetic force, f = - e (vd x B)
the principle that a positively charged ion can be :. Force on all the mobile electrons of the
conductor F = f· nA/ = - nAZe (Vd x B)
O

accelerated to high kinetic energy by making it


pass again and again smaller value of same or F = /(1x B) = IlB sine (2)

oscillating electrical field by making use of strong This is the expression for the force experienced
perpendicular magnetic field. Also, the frequency by current carrying wire.
of charge particle must be equal to the frequency
24. (i) If the particle is performing circular motion
of oscillating electrical field.
due to magnetic force, then
M

Expression for cyclotron frequency,


Centripetal force = Magnetic force
f=~ mv' . 0 mv
21tm (1) -- = qvB Sill 90 ~ r = -
r qB (1)
For schematic diagram Refer to Ans. 20. (1)
Time period is given by
Working Let initially positively charged is
accelerated towards D2 and enter into it. T = 21tr = 21t . mv = 21tm
v v qB qB (1)
Now, the charged particle experiences magnetic
Lorentz force due to a strong nonnal magnetic T = 21tm
field. It performs circular motion. The time taken qB
by the charge particle to complete half revolution 1 qB
Thus, frequency, f = - = --
is equal to half of time period of AC oscillator T .21tm (1)
between two dees.
146 o Chopterwise CBSE Solved Papers PHYSICS

(ii) Essential details of construction of 27. Refer to ADs. 20 (i) for diagram of cyclotron. (2)
cyclotron Cyclotron consists of Refer to ADs. 21 for working of cyclotron. (2)
(a) two semi-circular, hollow metallic D
Maximum KE of charged particle
shaped half cylinders known as dees.
(b) high frequency oscilla ting electric field is = q2B2r~
produced by AC oscillator. (having high 2m
frequency of AC voltage of few kVs) where, ro = radius of dees
(c) strong normal magnetic field is produced ., Radius of dees is limited, therefore, KGnax also

R
in dees using field magnetic. have limited value. (1)

(d) whole system is enclosed in high vacuum


28. (i) For figure of cyclotron
chamber. (1)
Refer to ADs. 20(i) and 21 for principle.
For working and schematic sketch Refer to
ADs. 20. (1)
For working of cyclotron Refer to ADs. 21.
.,'In case of the cyclotron, the particle moves on

SI
25. (i) Refer to ADs. 20 a circular path, the centripetal force required is
(ii) (a) Let the mass of proton = m; charge of provided by magnetic force, so magnetic
proton = q, mass of a-particle = 4m Lorentz force = centripetal force
Charge of a-particle = 2q
m; mv qBr
Cyclotron frequency, (1) qvB=- ~ r=-~ V=-
r qB m
v=l!!L ~ .;».
21tm m KE = ~ mv: = ~m (qBr)2= q2B2r2
22m 2m (2)
For proton frequency, vp oc i..
For maximum KE, r = ro (radius of dees). (1)
IT m
For a-particle, (il) Given, II = 2A, 12 = 5A, r = I cm = 1 x 10-2 m
Frequency, v oc~ Force per unit length between two wires.
IX 4m

or v oc-.!L ~ = ~ 21112 = !0-7 X 2 x 2 x 5 = 20 X 10-5


IX 2m
L 41t r lXl02
H
Thus, particles will not accelerate with same
cyclotron frequency. The frequency of proton is ~ = 2x 10-4 Nm' '
twice than the frequency of a-particle. (1) L rn
(b) Velocity, v = Bqr ~ voc i.. Currents flowing in wires are in opposite
m m directions, so the force will be of repulsive
For proton velocity, v i.. nature.
O

cc (1)
p m
29. Refer to ADs. 20. (5)
For a-particle,
30. For schematic sketch of cyclotron Refer to
Velocity, VIX cc ~ or VIX cc -.!L
4m 2m ADs. 20 (i). (1)
(3)
For working of cyclotron Refer to ADs. 21.
Thus, particles will not exit the dees with same
M

(i) Refer to ADs. 20. (1)


velocity. The velocity of proton is twice than the
(ii) The frequencies of charge particle must be
velocity of a-particles.
equal to the frequency of AC oscillator. This is
26. Force on the charged particle due to electric and known as resonance condition. (1)
magnetic field s, F = qE + q(v x B) This makes time period of charged particle and
For undeflected motion, oscillator equal. Therefore, the time taken by
F= 0 charge particle to complete half revolution is
qE+q(vxB)=O equal to the time of change the polarity of
~ E+ (v x B) = 0
dees. This facilitate the acceleration of charge
particle.
E = - (v x B) ~ I E I= I - vxB I If two frequencies do not match, then instead
E= vB sin I) of acceleration, charged particle may
where, I) = 90° accelerate. (2)
v=Em W
[TOPIC3] Magnetic Force and Torque between
Two Parallel Currents
3.1 Force on a Current Carrying 3.2 Torque Experienced by a
Conductor in a Uniform Current Carrying Loop in

R
Magnetic Field Uniform Magnetic Field
The magnetic force experienced by a (Magnetic Dipole)
current-carrying conductor placed in a uniform.
Torque experienced by a current-carrying loop
magnetic field is given by
placed in a uniform magnetic field B is given by

SI
F = I (1 x B) [vector form]
't = NIAB sin e
where, 1= a vector whose magnitude is equal to
length of the conductor and has identical direction where, e is the angle between the direction of
in the flow of electric current I and B = magnetic magnetic field B and that of vector N drawn
field. normal to the plane of the coil.
Magnetic force, F = IBI sin e or 't= Mx B
where, e is the angle between current and where, M = NIA
s
magnetic field.
IT F4
The direction of force is given by Fleming's left F3
hand rule.
P I

Magnetic Force between Two I B

Parallel Current Carrying Conductor


H
B
Magnetic force between two I
I 90°
straight parallel current-carrying
conductors is given by F1
B
~ = h.21112
O

L 41t r I, 1
~--~-!' Q I F2

Force will be of attractive nature, I I F3 = BII


I I

if direction of flow of currents are I


I
I
I

in the same direction i.e. parallel.


M

The force of repulsion will act when direction of


flow of currents are in opposite directions.
(antiparallel)
If both wires are oflength I} and 12 (if II > 12), then
magnetic force between two straight current
p
carrying wires/conductors is given by
F=h21112 12 F1 = BII
41t r smaUlength and M is known as magnetic dipole moment of
coil. Its SI unit is A_m2.
148 o Chopterwise CBSE Solved Papers PHYSICS

• The current sensitivity and voltage sensitivity of


3.3 Circular Current Loop as a galvanometer depends on number of turns of
Magnetic Dipole coil magnetic field B, area A of the coil and
torsion constant k of the' spring or suspension
A current loop behaves as a magnetic dipole. If we
wire.
look at the upper face, current is anti-clockwise, so
it has North polarity. If we look at lower face, then Restoring torque = Deflecting torque
current is clockwise. so it has South polarity so. ka=NIBA

R
upper face [Torque = Force x perpendicular distance

Qt5
't

= NIbB x a sin 90° = NIBA]


r Ii
I Current sensitivity, I = ~ = NBA
s I k

SI
Lower face
Its SI unit is rad/A or div/A,
Magnetic dipole moment of the loop, M = IA . . . e e Is NBA
Yo1tage SenSItIVIty, V = - = - = - = --
::::} IMI= I1tr Sv IRR kR
The magnitude of magnetic field on axis of a Its SI unit is rad/V or div/V
circular loop of radius r carrying stedy current I is Torque or moment of galvanometer
given by G =~= NBA
I k
for x» r
IT
Clearly I~ I
=~
Conversion of Galvanometer into an
If loop has N turns, M = NI A Ammeter and Voltmeter
H
A galvanometer can be converted into an ammeter
Magnetic Dipole Moment of by connecting a very low resistance (shunt S) in
parallel with galvanometer whose value is given by
a Revolving Electron
S= IsG
An electron being a charged particle, constitutes a
O

I - Is
current while moving in its circular orbit around
the nucleus, the magnetic dipole moment where, G = resistance of galvanometer
M=_e_nh Is = current through galvanometer
41tme I = total current in circuit and
M

where, me = mass of electron S = resistance of the shunt (low resistance).


h = Planck's constant Ammeter
r--------------------------------~
and n = 1,2,3, ... S

3.4 Moving Coil Galvanometer (I-Ig)

It is a device used to detect the current in


electrical circuit. It is based on the principle that a ~~------~ G r-----~~-
current carrying loop placed in a uniform I A Ig Ig B I
magnetic field experiences torque, the magnitude ---------------------------------
of which depends on the strength of current.
CHAPTER 4 : Moving Charges and Magnetism 149

Conversion of a Galvanometer to o 2 Marks Questions


Voltmeter 4. A square loop of side 20 cm carrying current
of lA kept near an infinite long straight
A galvanometer can be converted into voltmeter
wire carrying a current of 2A in the same
by connecting a very high resistance R in series
plane as shown in the figure.
with galvanometer which is given by 2A

R=~-G

~tOi'·

R
19

where, 19 = current through the


galvanometer
G = resistance of galvanometer and 20cm
=

SI
V potentials difference across the Calculate the magnitude and direction of
terminal A and B. the net force exerted on the loop due to the
Voltmeter current carrying conductor.
r--------------------------------,
,, All Indio 2015C
,,
,, 5. A rectangular coil of sider land b carrying a
,, current I is subjected to a uniform magnetic
A-----{ r-Jyy"y'Vv-----r'-.B ,, field B acting perpendicular to its plane.
,, Obtain the expression for the torque acting
,,
IT
on it. Delhi 2014C
I V I:
L ! 6. (i) Two long straight parallel conductors a
and b carrying steady currents Ia and
NOTE The resistance of an ideal ammeter is zero and an
Ib respectively are separated by a
ideal voltmeter is infinite. Ammeter is always
connected in series with electrical circuit, whereas
distance d. Write the magnitude and
direction, what is the nature and
H
voltmeter is connected in parallel with the circuit.
magnitude of the force between the two
conductors?
(ii) Show with the help of a diagram, how
the force between the two conductors

PREVIOUS YEARS' would change when the currents in


O

them flow in the opposite directions.


EXAMINATION QUESTIONS Foreign 2014

7. A coil of N turns and radius R carries a


TOPIC 3 current 1. It is unwound and rewound to
make a square coil of side a having same
M

o 1 Mark Questions number of turns N. Keeping the current I


same, find the ratio of the magnetic
1. Write the underlying principle of a moments of the square coil and the circular
moving coil galvanometer. CSSE Delhi 2016 coiL Delhi 2013C
2. Using the concept of force between two 8. A circular coil of closely wound N turns
infinitely long parallel current carrying and radius r carries a current I. Write the
conductors define one ampere of current. expressions for the following:
All Indio 2014 (i) The magnetic field at its centre.
3. Is the steady electric current the only (ii) The magnetic moment of this coil.
source of magnetic field? Justify your All Indio 2012
answer. Delhi 2013C
150 (23 ehapterwise eBSE Solved Papers PHYSICS

9. A steady current 11 flows through a long 15. A metallic rod oflength 1 is rotated with a
straight wire. Another wire carrying frequency v with one end hinged at the
steady current 12 in the same direction is centre and the other end at the
kept close and parallel to the first wire. circumference of a circular metallic ring of
Show with the help of a diagram, how radius r about an axis passing through the
the magnetic field due to the current 11 centre and perpendicular to the plane of
exert a magnetic force on the second the ring. A constant uniform magnetic field
wire. Deduce the expression for this force. B parallel to the axis is present

R
All India 2011 everywhere. Using Lorentz force, explain
10. How is a moving coil galvanometer how emf is induced between the centre
converted into a voltmeter? Explain giving and the metallic ring and hence obtained
the necessary circuit diagram and the the expression for it. Deihl 2013
required mathematical relation used. 16. A wire ABis carrying a steady current of

SI
All India 2011C 12A and is lying on the table. Another
11. A square coil of side 10 em has 20 turns wire CD carrying 5A is held directly above
and carries a current of 12 A. The coil is AB at a height of 1 mm. Find the mass per
suspended vertically and normal to the unit length of the wire CD, so that it
plane of the coil, makes an angle e with remains suspended at its position when
the direction of a uniform horizontal left free. Give the direction of the current
magnetic field of 0.80 T. If the torque, flowing in CD with respect to that in AB.
experienced by the coil equals 0.96 N-m, [Take, the value of g = 10 ms-2] All India 2013
IT
find the value of e. Oelhl2010C 17. A rectangular loopof wire of size
12. Define current sensitivity and voltage 2.5 em x 4 ern carries steady current of
sensitivity of galvanometer. Increasing 1 A. A straight wire carrying 2 A current
the current sensitivity may not is kept near the loop as shown. If the loop
necessarily increase the voltage and the wire are coplanar, find the (i)
sensitivity of a galvanometer, justify your torque acting on the loop and (ii) the
H
answer. All India 2009 magnitude and direction of the force on
the loop due to the current carrying wire.
13. Two long straight parallel conductor carry Delhl2D12
steady current 11 and 12 separated by a 1= 2A
distance d. If the currents are flowing in 2.5 em
O

the same direction, show how the


magnetic field set up in one produces an lA lA
attractive force on the other. Obtain the r
4em
expression for this force. Hence, define
one ampere. Deihl 2016 I
M

14. State the principle of working of a 1 em

galvanometer. A galvanometer of 18. Draw a labelled diagram of a moving coil


resistance G is converted into a voltmeter galvanometer and explain its working.
to measure upto V volts by connecting a What is the function ofradial magnetic
resistance Rt in series with the coil. If a field inside the coil? Foreign 2012
resistance R.z is connected in series with 19. Depict the magnetic field lines due to two
it, then it can measure upto V/2 volts. straight, long, parallel conductors
Find the resistance, in terms of Rt and R2, carrying currents 11 and 12 in the same
required to be connected to convert it into direction. Hence, deduce an expression for
a voltmeter that can read upto 2V. Also, the force per unit length acting on one of
find the resistance G of the galvanometer the conductors due to the other. Is this
in terms of B; and R2. All India 2015 force attractive or repulsive? Delhl2011C
CHAPTER 4 : Moving Charges and Magnetism 151

20. Find the expression for magnetic dipole


moment of a revolving electron. What is
o 3 Marks Questions
Bohr magneton? Delhi 2011 26. An electron of mass me revolves around a
21. State the underlying principle of working nucleus of charge +Ze. Show that it
of a moving coil galvanometer. Write two behaves like a tiny magnetic dipole. Hence,
reasons why a galvanometer cannot be prove that the magnetic moment

R
used as such to measure the current in a associated with it is expressed as
given circuit. Name any two factors on J.I. = - _e_L, where L is the orbital angular
which the current sensitivity of a 2m.
galvanometer depends. Deihl 2010 momentum of the electron. Give the
22. A moving coil galvanometer of resistance significance of negative sign. Deihl 2017

SI
G gives its full scale deflection when a 27. (i) Obtain the expression for the
current Ig flows through its coil. It can be cyclotron frequency.
converted into a ammeter of range (0 to I) (ii) A deuteron and a proton are
(I> I g) when a shunt of resistance S is accelerated by the cyclotron. Can
connected is converted into an ammeter of both be accelerated with the same
range 0 to 1, find the expression for the oscillator frequency? Give reason to
shunt required in terms of Ig and G. justify your answer. Oalhl2D17
Delhl2010C
IT
23. Write the expression for
the magnetic moment (m)
11 o 5 Marks Questions
due to a planar square 28. Explain, using a labelled diagram, the
loop of side I carrying a principle and working of a moving coil
steady current I in a
vector form. In the given

tJ galvanometer. What is the function of (i)


uniform radial magnetic field (ii) soft iron
H
figure, this loop is placed core?
in a horizontal plane Define the terms (i) current sensitivity
S R
near a long straight and (ii) voltage sensitivity of a
conductor carrying a steady current 11 at a galvanometer.
O

distance I as shown.
Why does increasing the current
Give reasons to explain that the loop will sensitivity not necessarily increase
experience a net force but no torque. voltage sensitivity? Deihl 2015. Foreign 2016
Write the expression for this force acting
on the loop. Deihl 2010 29. (i) Draw a labelled diagram of a moving
M

coil galvanometer. Describe briefly


24. Derive the expression for force per unit its principle and working.
length between two long straight parallel (ii) Answers the following questions.
current carrying conductors. Hence, (a) Why is it necessary to introduce a
define one ampere. Deihl 2009 cylindrical soft iron core inside the
25. Deduce the expression for the torque coil of a galvanometer?
experienced by a rectangular loop (b) Increasing the current sensitivity of
carrying a steady current I and placed in a galvanometer may not necessarily
a uniform magnetic field B. Indicate the increase its voltage sensitivity.
direction of the torque acting on the loop. Explain with giving reasons.
Foreign 2009 All Indio 2014
152 o ehapterwise eBSE Solved Papers PHYSICS

30. (i) State using a suitable diagram, the (ii) What is the importance of radial
working principle of a moving coil magnetic field and how is it
galvanometer. What is the function produced?
of a radial magnetic field and the soft (iii) Why is it that while using a moving
iron core used in it? coil galvanometer as a voltmeter, a
(ii) For converting a galvanometer into high resistance in series is required
an ammeter, a shunt resistance of whereas in an ammeter a shunt is
small value is used in parallel, used? All Indio 2010; Delhi 2009

R
whereas in the case of a voltmeter a 35. (i) Derive an expression for the force
resistance of large value is used in between two long parallel current
series. Explain, why? Deihl 2014C carrying conductors.
31. (i) Explain giving reasons, the basic (ii) Use this expression to define SI unit

SI
difference in converting a of current.
galvanometer into (a) a voltmeter (iii) A long straight wire B
and (b) an ammeter. AB carries a current
(ii) Two long straight parallel conductors I. A proton P travels I
p
carrying steady currents II and 12 are with a speed v, -d-rproton
separated by a distance d. parallel to the wire at
Explain briefly with the help of a a distance d from it in
v
suitable diagram, how the magnetic a direction opposite to A
IT
field due to one conductor acts on the the current as shown
other? Hence, deduce the expression in the figure. What is the force
for the force acting between the two experienced by the proton and what is
conductors. Mention the nature of its direction? All Indio 2010; Foreign 2008
this force. All Indio 2012
36. (i) Two straight long parallel conductors
32. A rectangular loop of size I x b carrying a carry currents II and 12 in the same
H
steady current I is placed in a uniform direction. Deduce the expression for
magnetic field B. Prove that the torque the force per unit length between
't acting on the loop is given by 't = m x B, them. Depict the pattern of magnetic
where, m is the magnetic moment of the field lines around them.
loop. All Indio 2012 (ii) A rectangular current carrying loop
O

EFGH is kept in a uniform magnetic


33. (i) Show that a planer loop carrying a
field as shown in the figure.
current I, having N closely wound
(a) What is the direction of the
turns and area of cross-section A,
magnetic moment of the
possesses a magnetic moment
current loop?
m=NIA.
M

(b) What is the torque acting on


(ii) When this loop is placed in a the loop maximum and zero?
magnetic field B, find out the
expression for the torque acting on it.
(iii) A galvanometer coil of 50 n resistance
)shows"full scale deflection for a
current of 5 mA. How will you convert
(r)
this galvanometer into a voltmeter of
range 0 to 15 V? Foreign 2011
34. (i) With the help of a diagram, explain
the principle and working of a
moving coil galvanometer.
Foreign 2010; Delhi 2009
o Explanations 6. (i) Let a and b be two long
straight parallel conductors. I.
a081

1. The principle of moving coil galvanometer is based


and Ib are the current flowing d
on the fact that when a current carrying coil is
placed in a magnetic field, it experiences a torque. through them and separated
by a distance d.
Torque produced is given by t = NIBAsina
Magnetic field induction at Q
[Symbols have their usual meaning) (1)
a point P on a conductor b
2. Refer to Text. due to current I. passing
3. Yes, the net magnetic force acting on a wire through a is

R
carrying a steady (constant) electric current I in BI = Jlo2I.
an external magnetic field B and is given 47td
F = IdlB (1) Now, unit length of b will experience a force
4. According to the question, as the loop is square, as F2 = ~Ib xl = Bllb f

its sides are parallel. So, force between two •• F2 = Jlo 2I.Ib

SI
parallel current carrying wires, 41t d
2A Conductor a also experiences the same amount
• of force directed towards b. Hence, a and b
1~ 1A attract each other. (1)

AO· ~ (ii) a®81

t,
B2®b

o C
20 em
IT
F = JloII 121
---4Q p
21tr
F1
Force on arm AB,
F _Jlox2Xlx20xlO-2
Now, let the direction of current in conductor b
AB - 21t X 10 X10-2 (1)
be reversed. The magnetic field B2 at point P
H
= 2Jlo N (Attractive, towards the wire) due to current I. flowing through a will be
1t downwards. Similarly, the magnetic field BI at
Force on arm CD, point Q due to current Ib passing through b will
F. _JloX2XlX20xlO-2 also be downward as shown. The force on a will
CD- 21tx30Xl02 be, therefore towards the left. Also, the force on
O

= 2Jlo N (Repulsive, away from the wire) b will be towards the right. Hence, the two
31t conductors will repel each other as shown. (1)

Force on arms BC and DA are equal and opposite. 7. Ratio of the magnetic moments
So, they cancel out each other. M, = 2INA,
Net force on the loop is F = F AB - FCD Me INAe
M

7
= Jlo [2-~]
1t 3
= 4Jlo
31t
= 4x41txl0-
31t _2(~r _1
-~-2
= 5.33 X 10-7 N (Attractive, towards the. wire) (1) (2)
- ,
5. Equivalent magnetic moment of the coil 8. (i) Magnetic field at centre due to circular current
m = IAn carrying coil, B = Jlo NI
.. m= Ilbn 2r (1)

where, n = unit vector.!.. to the plane of the coil. (ii) Magnetic moment, M = NIA = NI (1tr2)
:.Torque =mxB=Ilb(nxB) =0 (2) M = 1t utr' (1)

As nand B are parallel or anti-parallel to each where, r is the radius of circular coil, Jlo is
other. permeability of free space and N is number of
turns.
154 o Chapterwise eSSE Solved Papers PHYSICS

9. In these types of questions, we are calculating force 10. The resistance of an ideal voltmeter is infinity or very
on a wire in the field produced by the other current high in practical condition. So, to convert a
carrying wire. galvanometer into voltmeter, its resistance needs to
be increased, which can be done by a high
To find expression of force between two parallel resistance in series connection with it.
wires.
Let two infinitely long straight current carrying A galvanometer can be converted into a voltmeter
conductor carry currents I) and 12 in the same by connecting a very high resistance R in series
direction. with it. (112)

R
Let R is so chosen that current I9 gives full
ALt Magnetic field B) due to first wire on seconds, i.e. deflection in the galvanometer where I9 is the
range of galvanometer.
(112)
,Voltmeter
B) = ~o 2l) = ~OI) ... (i) r---------------------.

SI
41t r 21tr (112) I I

":)f I
:[g R:
The magnetic field is perpendicular to the plane of I G I
~
paper and directed inwards i.e. (X) type. A : : B
t
I
1I

::I
1+--- V -----+l (112)

•• Let galvanometer of resistance G, range I9 is to be


converted into voltmeter of range V (volt). Now,
[2 V V
x x
V=Ig(G+R) ~R+G=- ~R=--G
Ig Ig
IT
[
••
..
••
r
F2 x x

x
x

x x
xx 1
L The appropriate scale need to be graduated
measure potential difference.
11. Here, Area (A) of coil
= 10 x 10 = 100 cm2 = 10-2 m2
Number of turns, N = 20 turns
to
(1)
H
•• x x Current, I = 12 A
Number of the coils make angle with magnetic
field = a =?
Now, magnetic force on length L of second wire is Magnetic field, B = 0.8T
given by (1/2) Torque, 't = 0.96 N-m
O

(112)

F2 = I~) L sin 90° .: Torque ('t) experienced by current carrying coil,


the magnetic field is
F2 = 12 (~O.
41t
2l) ) L
r
't = NIABsina (1)

0.96= 20 x12xlO-2 x 0.8 x sin a


~ F2 = ~o . 2l) 12 = ~OI)I2 ... (ii)
M

L 41t r 21tr sin a = 0.96 =~ ~ a=~ rad


1.92 2 6 (1/2)
By Fleming's left hand rule, the direction of force
F2 is perpendicular to the second wire in the plane 12. Current sensitivity The deflection produced in
of pape~ towards the first wire. the coil of galvanometer per unit flow of electric
! 11 1 q J ""1
Similar y, I , •

magnetic force on 1st wire is given by current through it, i.e.


,.F fi.=~o.2I)I2 ... (iii) Current sensitivity,
L. 41t r I = ~ = NBA
9 I K
F)=-F2
The force Fi is directed towards the second wire. where abbreviations are as usual. (1/2)

Thus, two straight parallel current carrying Voltage sensitivity The deflection produced in
conductor have the same direction of flow of the galvanometer is per unit applied potential
currents attracting each other. (112) difference across it.
CHAPTER 4 : Moving Charges and Magnetism 155

..
. ..
V 0Itage sensinvny. v: = -e = -e = --NBA A high resistance that is connected in series with
s V IR K.R the galvanometer to convert into voltmeter. The
where abbreviations are as usual.Its unit is rad I A value of the resistance is given by (1)
or div I A. (112)
R=~-G
Increasing the current sensitivity may not Ig
necessarily increase the voltage sensitivity,
where, V = potential difference across the
because the current sensitivity increases with the
terminals of the voltmeter, Ig = current through
increase of number of turns of the coil but the
resistance of coil also increases which affect the galvanometer and

R
G = resistance of the galvanometer.
adversely on voltage sensitivity. (1)
When resistance R] is connected in series vy'tf" the
13. To find the force on a current-carrying wire due to a galvanometer, then
second current-carrying wire, first find the field due V
R] =- -G ...(i)
to second wire at the site of first wire. Then, find the
Ig

SI
force on the first wire due to that field.
When resistance R2 is connected in series with the
Let us consider AI BI and galvanometer, then
A2B2 are two infinite long B, B2 V
R2 = - - G ... (ii)
straight conductors.
II and 12 are the currents B2 B, »,
flowing through them and From Eqs. (i) and (ii). we get
they are r distance apart.
F, F2
P
R] - R2 = -V and G = RI - 2 R2
Magnetic field induction at 2Ig
I, 12
a point P on the conductor
IT
The resistance R3 required to convert the given
A2B2 due to current II A, A2 galvanometer into voltmeter of range 0 to 2V is
passing through AI BI is
B] = ~o2l] given by R3 = 2V - G
Ig
4~r n)
The unit length of A2B2 will experience a force as ~ R3 = 4 (R] - R2) - (2R1 - R2) = 3R] - 2R2
G in terms of RI and R2 is given by
H
P2=BI12Xl=B]I2
or P = ~o . 2l]12 G = RI - 2R2 (1)
2
4~ r
15. Suppose, the length of the rod is greater than the
Conductor AI BI also experiences the same amount radius of the circle and rod rotates anti-clockwise
of force directed towards the wire A2B2.
and suppose, the direction in the rod at any
O

Therefore, force between two current-carrying instant be along + Y-direction. Suppose, the
parallel conductor per unit length is direction of the magnetic field is along
P = h. 2l]12 + Z-direction.
4~ r Then, using Lorentz law, we get the following
Two linear parallel conductors carrying-currents in
F= - e (v x B) ~ F = - e (v j x B~ ~ F= - evBl (1)
the same direction attract each other while in
M

opposite direction they repel each other. Thus, the direction of force on the electrons is
Definition of One Ampere along X-axis.
1 ampere is the current which flows through each Thus, the electrons will move towards the centre,
of the two parallel uniform long linear conductors, i.e. the fixed end of the rod., c . - •

which are placed in free space at a distance of 1 m This movement of electrons wili'ieshff Ji\urrent
from each other and which attract or repel each and hence, it will produce emf in the ~od Between,
other with a force of 2 x 10-7 Nm-] of their the fixed end and the point touching the ring.
lengths. (1) Let e be the angle between the rod and radius of the
circle at any time t.
14. According to the principle of working of a moving
coil galvanometer, when a current carrying coil is Then, area swept by the rod inside the circle
placed in a magnetic field, it experiences a
= ~ ~ril
torque. 2
156 o Chapterwise CSSE Solved Papers PHYSICS

Now, induced emf = B x :!... (~ 1tr1l) (ii) Net force on loop,


dt 2 F=Fi -F2
[towards straight conductor]
= ~1tr2Bde
F = 8 X 10-7 - 3.55 X 10-7
2 dt
F= 4.45 x 1O-7N
= ~ 1tr2Bw
2 The forces on two branches of loop are equal in
magnitude and opposite in the directions, hence
= ~ 1tr2B(21tv)

R
2 they balance each other. (112)

= 1t2r2Bv 18. Moving Coil Galvanometer Principle


(1)
noi -o'J Its working is based on the fact that when a
NOTE There will be an induced emf between the two ends of current carrying coil is placed in a magnetic field,
the rods also. it experiences a net torque.

SI
Scale
16. Anti-parallel currents repel.
The magnetic force of repulsion on the upper wire
should be balancing its own weight.
For wire CD to remain suspended at its position in
equilibrium,
Permanent magnet
magnetic force on CD due to AB = weight of CD
.. ~Olj 121 = mg (g = 10m/s2)
T
21tr
IT
m=2xlO-71jI2 (forl=lm)
rg
2xlO-7 xizx 5
m=--,----
10 3 xlO
10-3 kgm "
H
= 1.2 X

Current in CD should be in opposite direction to


that in AB.
17. There will be force of attraction between the
straight wire and 4 cm long arm of loop nearer to
O

the straight conductor. Phosphor


bronze
Fi =~o 2x2xl x(4XlO-2)
strip
41t (2xlO 2)
T1 T2
[towards straight conductor]
Fi = 8 x 1O-7N ... (i) (1)
M

Similarly, force on other 4 ern arm of loop, away


from the straight conductor
F2 = ~o x 2x2xl x (4 X 10-2)
41t ("4.5x 10 2)
F2 = 3.55 x 1O-7N ... (ii)
1 [away from conductor]
(i) Since, Fi and F2 are of different magnitudes,
therefore, do not form couple and hence
Torque, t = 0 (112)
(1/2)
CHAPTER 4 : Moving Charges and Magnetism 157

Working 19. For deduction of the force acting between two


Suppose, the coil PQRS is suspended freely in the long parallel conductors Refer to Ans. 8 (i). (1)
magnetic field.
Let I =length PQ or RS of the coil
b = breadth QR or SP of the coil
n = number of turns in the coil
Area of each turns of the coil, A = I x b

R
Let B = strength of the magnetic field in which
coil is suspended.
Magnetic field lines due to both conductors
I = current passing through the coil in the
direction of PQRS Current-carrying conductors having same direction
Let at any instant of time, a be the angle which the of flow of current, so the force between them ~

SI
normal drawn on the plane of the coil makes with be attractive. (1)
the direction of magnetic field. The rectangular 20. As electric current associated with the revolving
current carrying coil when placed in the magnetic
electron
field experiences a torque whose magnitude is
given by 't = NIBA sin a (112) l=~=~
T 21tr
Due to this deflecting torque, the coil rotates and
suspended wire gets twisted. A restoring torque is where, time period T = 21tr
set up in the suspension wire. v
Let 9 be the twist produced in the phosphor bronze r = radius of orbit
strip due to rotation of the coil and
IT v = velocity of electron
k be the restoring torque per unit twist of the The magnetic moment due to the current,
phosphor bronze strip.
Then, total restoring torque produced = k9 ~ = LA = ~ x 1tr2 ~ ~ = evr
21tr 2 (1)
In equilibrium position of the coil,
If electron revolves in anti-clockwise sense, the
Deflecting torque = Restoring torque current will be in clockwise sense. Hence, according
H
.. NIBA = k9 or I = _k_9 = G9 to right hand rule, the direction of magnetic
NBA moment must will be perpendicular to the plane of
where, _k_ = G orbit and directed inwards to the plane.
NBA evtm el
So, ~ =--=-
[constant for a galvanometer] On On
O

It is known as galvanometer constant. where, vrm = I = angular momentum orbital of


electron and in vector form,
• Current sensitivity of the galvanometer is the
I
deflection per unit current. ~=-e-
On
~=NAB
Negative sign indicates !.l and I are in mutually
I k
opposite directions. From Bohr's postulates,
M

• Voltage sensitivity is the deflection per unit nh


1= mvr = -, where n = 1, 2, 3, ...
voltage.

* N:S (i)
= = N:S i
The uniform radial magnetic field keeps the plane
[','V=IR]
21t
e nh
~ = 2m' 21t = n!.lInin
(1)

eh
of the coil always parallel to the direction of the where, !.l . = -- called Bohr's magneton.
mm 47tm
magnetic field, i.e. the angle between the plane of
the coil and the magnetic field is zero for all the 21. Principle The current carrying coil placed in
orientations of the coil. (1) normal magnetic field experiences a torque which
is given by
't =NIAB
158 IZI ehapterwise eBSE Solved Papers PHYSICS

where, N = number of turns 23. The magnetic moment of a current carrying loop
I = current m=IA
A = area of coil where, A = area of the loop (square)
B = magnetic field .. A = /2 n
The galvanometer cannot be used to measure the n
Here, is a unit vector normal to the direction of
current because area vector.
(i) all the currents to be measured passes through The forces acting on the arms QR and SP of given
coil and it gets damaged easily as hair line

R
(in question figure) loop are equal, mutually
spring or opposite and collinear. Hence, they are balanced by
(ii) its coil has considerable resistance because of one another. (1)
length and it may affect original current. Force on arm PQ, Fi = B II = _,.. "I I
"I_0 _I II = _,.._0_1-
['12-2=1) 21t/ 21t
Current sensitivity of galvanometer can be Obviously, Fi is of the attractive nature and

SI
increased by directed towards MN.
(i) increasing the magnetic field Again, force on arm RS,
(ii) decreasing the value of torsional constant. (1) F2=B2Il=J.1.01IIl = J.1.oII I
21t(2/) 41t (1)
22. The resistance of an ideal ammeter is zero or very
low in practical condition, so to convert a
F2 is perpendicular to wire RS and directed away
galvanometer into ammeter its resistance needs to from the conductor MN.
be decreased which can be done by connecting a ,'. Net force on loop PQRS,
low resistance in its parallel order. :::) Fnet=~-F2
= J.1.0III
IT _ J.1.0II I
A moving coil galvanometer of range I9 and
resistance G can be converted into ammeter by 21t 41t
connected very low resistance shunt in parallel or F - J.1.0Il I [attractive]
net-~
with galvanometer.
.,' To convert a galvanometer into an ammeter, As, FI and F2 are collinear, hence does not produce
shunt resistance is connected in parallel with the torque on the loop PQRS.
H
galvanometer, so the potential difference across the 24. Force between two straight parallel current
combination is same. carrying conductors
:. PD across galvanometer = PD across shunt S
F = &. 2/112
IgG = Is S (1) 41t r
But Is + Ig = I Let II = 12 = 1 A. r = 1rn,
O

:::) Is =I- 19 then F = 10-7 X 2 (1) (1) = 2 x 10-7


:::) IgG=(I-1g)S (1) (1)

:::) S= IgG One ampere It is the value of steady current


1- 19 (1) which flows through two straight, parallel
infinitely long current carrying conductors of
M

The shunt resistance S to be connected to convert negligible cross-section placed in air or vacuum at
galvanometer into an ammeter. a distance of 1 m and they experience a force of
Ammeter attractive or repulsive nature of magnitude
s 2x 10-7 N/mon their unit length. (2)

25. Let a current carrying rectangular loop PQRS


(I - Ig) (I - Ig) carrying a steady current I placed in a uniform
magnetic field B keeping axis of the coil
1 I
---;-4~~G~--~r--- perpendicular to field as shown in figure. Let at
A Ig B any instant the area vector A makes an angle e
with the direction of magnetic field B.
CHAPTER 4 : Moving Charges and Magnetism 159

same line of action. Therefore, they balance each


p other. (cancel out)
Now, torque due to Fj and F2 is given by
: Q t = force x perpendicular distance between lines of
I F1 i
I
action of Fj and F2•
I
I
t=Fxbsina
l-!
I
I
I But, Fj = F2 = F = IBI ~ t = (IBI) x (b sin 9)
I

B !
I
t = IB (Ib) sin a
t = IBA sin a

R
I
I B
I

! F2 where, A = Ib = area of coil for N turns of coil


I
I
I
t = NIAB sin a (1)

S t-:::::::: i I 26. When electron revolves around a nucleus, it


creates circular current around it. In this way, it

SI
b~R
is equivalent to a current carrying coil. So, it
F3 behaves as a tiny magnetic dipole. (lJ
Fig. (0) We know that a current carrying coil behaves like a
magnetic dipole having dipole moment equal to
i A, where i is current and A is area of the coil.
i = :...= ero = eMer2ro ... (i)
m T 21t 21tmr2
Iup I where, e is charge of electron. tn
I
I
Angular momentum of electron is L = M,r2ro
I
I
IT
I
I Substituting the above in Eq. (I), we get,
L___________________ Idown . eL
b sin e Z=---
27tM,r2
2
Hence, jJ.= iA = eL1tr =~
Fig. (b) 27tM,r2 2M,
H
Let length and breadth of coil are 1 and b, -eL
In vector form, jJ.=-
respectively. 2M, rn
Now, magnetic force on PS arm of the coil is given
Here, negative sign indicates jJ.is directs away from L.
by 1) = IBI sin 90°
[.,' PS II axis of coil.:', 9 = 90°] 27. (i) In the presence of the magnetic field, the
proton moves in the circular path whose
O

Fj=IBI ... (i)


radius is r, which depends on its speed.
By Fleming'S left hand rule, the direction of force is
perpendicular to SP and B is along upward Hence,
direction. Similarly, force of QR arm of the coil. magnetic force on q = centripetal force on q.
2
F2 = IBI sin 90° ... (il) ~ qv B sm mv
. 900 =-- m;
~ qv B =--
The direction of force is perpendicular to QR and B r r
M

is along downward direction. sin900= 1)


mv
.,' 1) and F2 are equal in magnitude, opposite in ~ r=-
direction, parallel to each other acting on the loop qB m
forms a couple which try to rotate the coil. (1)
Time period of proton is, T = 21tr = 21tm
Now, force on RS part of the coil v qB
F3 = IBb sin (90° + 9) ., Frequency, I=.!... = qB ... (i)
F3 = IBb CDS a T 21tm
and force on PQ part of the coil The frequency of oscillation of the charged
F4 = IBb sin (90° - 9) = IBb cos a particle from the above expression is
qB
But Fleming'S left hand rule, F3 and F4 are equal in
magnitude and opposite in direction along the
lost: = 21tm

It is also known as cyclotron frequency. (lJ


160 o Chapterwise CBSE Solved Papers PHYSICS

(ii) Let the mass of proton", m It is known as galvanometer constant, i.e. I oc O.It
Charge of proton", q, Mass of deuteron= 2m means that the deflection produced is
proportional to the current flowing through the
Charge of deuteron e q
galvanometer. (1)
Cyclotron frequency, v "'~ ~v oc!L As, we can see in the figure given that
21tm m
Cylindrical soft iron core which not only makes
For proton frequency, vp oc!L ... (i) the field radial but also increases the strength of
m
the magnet.
For deuteron frequency, Vd oc!L ... (ii)

R
2m Radial magnetic fields is a field in which coil of
the galvanometer always remain parallel to the
From Eqs. (i) and (ii), we get, vp '" 2vd field even on large deflection.
Thus, frequency of proton is twice that of
Current sensitivity It is defined as the
deuteron.
deflection produced per unit current flowing
No, both cannot be accelerated with same through the galvanometer. It is given by

SI
oscillator frequency as they have different mass.
(1)
o NAB
Is "'-"'--
I k
28. Principle
Its working is based on the fact that when a Voltage sensitivity It is defined as the
current carrying coil is placed in a magnetic field, deflection produced per unit voltage
it experiences a torque.
For Fig. refer to Ans. 18.
(1/2)
~ Vs'" % '" (N~)i ~ Vs'" N~ . :R
Let at any instant, a be the angle which normal ~ v'" NAB
IT
drawn on the plane of the coil makes with the kR (1)

direction of magnetic field, the rectangular coil Current sensitivity does not depend upon
carrying current when placed in the magnetic resistance (I?), whereas voltage sensitivity does, as
field experiences a torque. Whose magnitude is evident from their expression. Current sensitivity
given by t '" NIBAsina can be increased by increasing the number of
Due to deflecting torque the coil rotates and turns of the coil. However, this increase the
resistance of the coil, since voltage sensitivity
H
suspension wire gets twisted. A restoring torque
is set up in the suspension wire. (1'10) decreases with increase in the resistance of the
coil the effect of increase in number of turn is
Let 0 be the twist produced in phosphor bronze
nullified in the case of voltage sensitivity (1)
strip due to rotation of the coil and k be the
restoring torque per unit twist of the phosphor 29. (i) Refer to Ans. 18. (1)
(ii) (a) It is necessary to introduce a cylindrical
O

bronze strip.
soft iron core inside the coil of a
Then, total restoring torque produced", kO
galvanometer because magnetic field is
In equilibrium position of the coil, increased, so its sensitivity increases and
rr Deflecting torque", Restoring torque magnetic field becomes radial. So, angle
between the plane of coil and magnetic line
NIBA '" kO ~I '" _k_O '" GO
M

of force is zero in all orientations of coil. (2)


NBA
(b) The relation between current sensitivity
where, _k_ '" G and voltage sensitivity is given by
NBA
t· Voltage sensitivity Current sensitivity
JIl Resistance of coil
Is
Vs "'-
Rc (1)

If Rc is constant, then Vs oc Is
This means if Vs increases, Is also
increases.
But, if Rc increases in the given ratio, then
Vs is a constant. (1)
CHAPTER 4 : Moving Charges and Magnetism 161

30. (i) Refer to Ans. 18. (2) 32.


(ii) Refer to Ans. 22 (i). (3) (112)
31. (i) A galvanometer of range Is and resistance G1
can be converted into
(a) a voltmeter ofrange Vby connecting a high
resistance R in series with it where value is
given by R = !.- G
Is

R
(b) an ammeter of range I by connecting a very
(1)
low resistance (shunt) in parallel with
galvanometer whose value is given by
--~----------------B
S = IsG A

I - Is (1)

SI
(ii) Let two straight wires of infinite length are
carrying currents, II and 12 in the same
direction and separated by distance d apart
from each other. (1) [1)
The magnetic field due to wire 1 at any point
on wire 2, The magnetic force on Be and DA part of wire are
equal in magnitude, opposite in direction along
B1 = Ilo 21J ••• (i)
47t d the same line. Therefore, they balance each other.
(112)
The distance of Bl is perpendicular to plane of
IT
paper and directed inward. Let at any instant area vector of coil made an
angle a with the direction of magnetic field.
CD I
I
® I
I
:.1) and F2 form couple which try to rotate the
I I
coil.
• •
From figure,
• • x x
:. Torque, t = force x arm of the couple

T • •
H
= (IBI) x MD
• • x x
= IBI x bsin a = IB (lb) sin a
L • •
11
x x
12
t = IB Asin a (1)

1
• • x x
where, A = Ib = area of coil
d
• • x x
O

t = IABsina
• • F1 F2 x x
B But, m=IA
• • x x

t = mBsina
• • x x ..
In vector form, t = mx B (1)
(112)
M

33. (i) Torque on rectangular loop,


Magnetic force on wire 2 in L length of it
t = NIAB sin a ... (i)
F2 =12 Bl L sin 90°= 12 (IlO. 211)L xi
where, symbols are as usual.
47t d (1/2)
Also, torque experienced by magnetic dipole of
.. F2 = Ilo 21112 [towardswire] ... (ii) moment m are placed in a uniform magnetic
47t d field.
By Fleming'S left hand rule.
t = mBsina ... (ii)
Similarly, force on wire 1 due to wire 2 can be
Comparing Eqs. (i) and (ii), we get
proved 1) = Ilo 21112 ... (ill) The magnetic dipole moment,
47t d
m=NIA
Thus, the nature of force is attractive.
Also, m is along A. ~ m = NIA [2)
When direction of flow of current gets in opposite (ii) Refer to Ans. 21. (1/2)
direction, the nature of force becomes repulsive. (2)
162 o ehopterwise eBSE Solved Popers PHYSICS

(ill) G=50o.,Is=5X1O-3AV=15V (ii) In radial magnetic field, the plane of the coil
+__v-----+ is always parallel to the plane of the magnetic
•.. ----- -- -----------,
I I field and area vector of coil is perpendicular to
~ •..•~ •.~high resistance magnetic field. It is always exerts maximum
~yyyy+-
Ig i i torque on the coil. (1)
I I
" 1 I
_______________ - II
(ill) The voltmeter connected in parallel with the
Voltmeter electrical circuit elements to measure
1 potential difference. For exact measurement
V=ls(G+R)
of PD voltmeter must draw minimum current

R
r/;i#, .,,',b ,,[ R=~-G
19
I
,
which is possible only when it has high
,'f -+<Jq+l.-) "!:~ _ resistance. Ammeter is connected in series
with the electrical circuit and current to be
~ _1_5__ 50 ~ R = 29500.
5 x 10-3 measured passes through it.
(1)
In order to protect the galvanometer, a feeble

SI
A resistance R = 29500. is to be connected in series current must pass through the galvanometer,
with galvanometer to convert it into a desired it is possible only when a low resistance
voltmeter. (1'10) (shunt) is connected in parallel with
34. (i) Refer to Ans. 18 for principle. (112) galvanometer to allow the major part of the
H current to pass through it. . (2)
35. (i) Refer to Ans. 19 (ii). (2)
Phosphor (ii) As, ~ =!-Lo 2l, l2
bronze L 41t r
strip
IT IJ = [2 = 1 A, r = Irn
T2
~ = 2x1O-7 Nm-'
L
For definition Refer to Ans. 20. (1)
(ill) Here, magnetic field due to the current
carrying conductor at a distance d from it is
given by
H
B =!-Lo 2l
41t d (1/2)
. . Force on proton,
f!I
F = (e) (0 B sin 90°
O

~ F= evB
(112)
F = ev (!-Lo 2l)
The coil remains suspended in radial magnetic 41t d
field so that it always experiences maximum F = !-Lo . 2lev
.n .(
torque. 41t d (1)
When current passes through the coil,
M

The proton is directed perpendicular to


deflection torque 't (9) is produced given by
straight conductor and away from it. (1/2)
'tdeflection = NIAB sin 90° ... (i)
As a result, coil rotates and phosphor bronze 36. (i) Refer to Ans. 19. (3)
strip gets twisted. As a result restoring torque (ii) (a) Perpendicular to the plane of the paper
given by and directed inward. (1)

't restoring = k9 ... (ii) (b) When angle between area vector of coil
where, k = torsional restoring constant and magnetic field is 90°, then maximum
. . In equilibrium, torque experienced by the coil.
(deflecting = 't restoring ~ NIAB = k9 When 9 = 0° or 180°, then torque will be
minimum, i.e. zero. (1)

l = (:AB)9~ I ac9
greater the current, greater the deflection. (1)
Value Based Questions (From Complete Chapter)
04 Marks Questions 2. Mr. Sharma a 65 year old person often
complained of neck pain. One day his
1. Kamal's uncle was advised by his doctor grandson Mridul, suggested that magnetic
to undergo an MR1 scan test of his chest therapy is very effective in reducing pain.
and gave him an estimate of the cost. Not He said that the permanent magnet!
knowing much about the significance of electromagnet, used in the device will

R
this test and finding it to be too expensive help to produce joule's heating effects in
he first hesitated. When Kamal learnt the blood stream, which helps the blood
about this, he decided to take help of his flow better. He immediately contacted his
family, friends and neighbours and friend in Chennai who was running a
arranged for the cost. He convinced his magnetic therapy clinic.

SI
uncle to undergo this test so as to enable Read the above passage and answer the
the doctor to diagnose the disease, he got following questions.
the test done and resulting information (i) What values did Mridul exhibit
greatly helped the doctor to give him towards grandfather?
proper treatment. (ii) What is the S1 unit of magnetic field?
(a) What according to you, are the values (iii) Give the magnitude of magnetic field
displayed by Kamal? in 4 A current carrying circular coil or
(b) Assuming that the MR1 scan test radius 2 cm.
IT
involved a magnetic field of 0.1 T,
Ans. (i) He is of responsible behaviour, concern and
find the maximum and minimum awareness. (1)
values of the force that this field (ii) The 51 unit of magnetic field is Tesla. (1)
could exert on a proton moving with ... ) B _ ~oI _ 41t xur ? X 4 _ 50.24 10-5
a speed of104 ms-I. State the ( III --- ---x
]r ·2 x 2 x 10- 2 4
condition under which the force can
= 12.56 x 10-5 T
H
be minimum. Foreign 2013 (2)

Ans. (a) Values displayed by Kamal: 3. Ms. Kanchan is a student of PG course in


(i) Being educated, he knows about MRI nanotechnology lab in lIT Kanpur. The
(magnetic resonance imaging}.
first day, when she went to the lab she
(ii) Took prompt decisions to take the help of
met Mr. Cobra, the lab assistant. He
O

his family, friends and neighbours to


arranged the cost of MRI. greeted her and advised her not to touch
(iii) He showed his empathy, helping attitude the wires, which were suspended from the
and caring nature for his uncle. (2) roof at every part of the lab as they were
(b) Magnetic force on moving charge particle in from high voltage lines. He also told her
uniform magnetic field 8 can be given as not to bring any of the two wires closer to
M

F = q(v x 8) or IFI= qvBsine each other during any experimental


(i) Maximum force at e = 90° applications. He helped her in
F = qvB= 1.6 X 10-19 X 104 x 0.1 understanding about precautions that has
= 1.6 x 1O-16N [1] to be taken in the lab.
(ii) Minimum force at e = 0° and 180° Read the above passage and answer the
following questions.
F=O
i.e. force is minimum when the charge (i) What values did Mr. Cobra exhibit?
particle either move paraliel or Give any two.
anti-parallel to the magnetic field lines. [1] (ii) Why should not the two high voltage
wires be brought close to each other?
164 o ehopterwise eSSE Solved Papers PHYSICS

Ans. (i) He is of responsible behaviour, sensitivity, v


V=-
concern for others. [2] 27tr
(ii) Because the two wires have high magnetic = 2x106s-1
field around them. [2]
= 2x106 Hz
4. Alka and her sister were watching a =2MHz [1]
movie in which the phenomena of aurora
borealis was shown. Alka could not Energy, (E) = ~m;
2

R
believe her eyes that, such a colourful
= ~ x 9 X 10-31 X 9 X 1014
display like the one during commonwealth 2
games could be created by the nature. She = 40.5 X10-17 J
went to the library, but could not find the = 4 x1O-6J
right book. So, she consulted her teacher

SI
= 25keV [1]
who guided her. Hence, Alka understood
that during a solar flare, a large number 5. In the birthday party of Kamal, his
of electrons and protons are ejected from parents gave big slinkie to all his friends
the sun. Some of these get trapped in the as a return gift. The very next day, during
earth's magnetic field and move in a the physics class Mr. Mohan, the teacher
helical path along the field lines. As the explained them about the production of
density of the field lines increases near magnetic field using current carrying coil
the poles, these particles collide with and also said that they can make
IT
atoms and molecules of the atmosphere permanent magnet, using such coils by
emitting green and pink light. Alka passing high currents through them. That
shared this knowledge with her class night Priyanshu, a friend of Bharat, asked
when they studied the chapter of moving his father about the coils, and their shape.
charges in magnetic field. His father asked him to bring the slinky,
Read the above passage and answer the that his friend gave and explained the use
H
following questions of toroid and solenoid.
(i) What values did Alka have? Read the above passage and answer the
(ii) What is the radius of the path of following questions
an electron moving at a speed of (i) What value did Priyanshu's father
O

3 x 107 mls in a magnetic field of have?


6 gauss perpendicular to it? What is (ii) What is the difference between
its frequency? Calculate its energy in solenoid and toroid?
kilo electron volt.
(iii) Give the value or magnitude of
Ans. (i) She has analytical knowledge, nature of magnetic field in solenoid.
M

appreciations, diligence, curiosity, presence of


Ans. (i) Priyanshu's father is responsible and
mind. [1]
knowledgeable, makes his child understand the
(ii) Using equation,
concepts of solenoid and toroid. [1]
mv2 (ii) A solenoid has magnetic field straight within
qvB=-
r the turns and in toroid, it is in form of
mv concentric circles. [1]
r=-
qB (iii) The magnitude of magnetic field in solenoid is
9 x 10-31 X 3 X 107 given by
1.6x 10 19X 6x 10 4 B = lJ.onI
= 2.8125X10- 1 [where, n = number of turns/length,
I = current in coil] [2]
= 28.125 ern [1]

You might also like